Tag Archives: 2016

ĐỀ THI CHỌN ĐỘI TUYỂN HSG QUỐC GIA CỦA TRƯỜNG PTNK NĂM 2016 – 2017

ĐỀ THI

Ngày thi thứ nhất

Bài 1. Tìm tất cả $a$ để dãy số $\left(u_n\right)$ hội tụ, biết $u_1=a$ và $\forall n \in \mathbb{N}^*$ thì:

$\quad\quad\quad\quad\quad\quad\quad\quad\quad\quad u_{n+1}=\left\{\begin{array}{l}2 u_n-1 \text { nếu } u_n>0 \\ -1 \text { nếu }-1 \leq u_n \leq 0 \\ u_n^2+4 u_n+2 \text { nếu } u_n<-1\end{array}\right.$

Bài 2. Tìm số nguyên dương $k$ nhỏ nhất để bất đẳng thức

$\quad\quad\quad\quad\quad\quad\quad\quad\quad\quad x^k y^k z^k\left(x^3+y^3+z^3\right) \leq 3$

luôn đúng với mọi số thực dương $x, y, z$ thoả mãn điều kiện $x+y+z=3$.

Bài 3. Cho hàm số $f: \mathbb{N}^* \rightarrow \mathbb{N}^*$ thoả mãn hai điều kiện sau:

$\quad\quad$ i) $f$ là hàm tăng thật sự trên $\mathbb{N}^*$.

$\quad\quad$ ii) $f(2 n)=2 f(n) \forall n \in \mathbb{N}^*$.

(a) Giả sử $f(1)=3$ và $p>3$ là số nguyên tố. Chứng minh rằng tồn tại số nguyên dương $n$ sao cho $f(n)$ chia hết cho $p$.

(b) Cho $q$ là số nguyên tố lẻ. Hãy xây dựng một hàm $f$ thoả mãn các điều kiện của bài toán mà $f(n)$ không chia hết cho $q$ với mọi $n$ nguyên dương.

Bài 4. Cho tam giác $A B C$ có góc $\angle B A C$ tù và $A H \perp B C(H$ nằm trên $B C)$. Điểm $M$ thay đổi trên cạnh $A B$. Dựng điểm $N$ sao cho $\Delta B M N \sim \triangle H C A$, với $H$ và $N$ nằm khác phía đối với đường thẳng $A B$.

(a) Gọi $C M$ cắt đường tròn ngoại tiếp tam giác $B M N$ tại $K$. Chứng minh rằng $N K$ luôn đi qua một điểm cố định.

(b) Gọi $N H$ cắt $A C$ tại $P$. Dựng điểm $Q$ sao cho $\triangle H P Q \sim \triangle H N M$, với $Q$ và $M$ nằm khác phía đối với đường thẳng $N P$. Chứng minh rằng $Q$ luôn thuộc một đường thẳng cố định.

Ngày thi thứ hai

Bài 5. Với mỗi số nguyên dương $n$, tồn tại duy nhất số tự nhiên $a$ thoả mãn điều kiện $a^2 \leq n<(a+1)^2$. Đặt $\Delta_n=n-a^2$.

(a) Tìm giá trị nhỏ nhất của $\Delta_n$ khi $n$ thay đổi và luôn thoả mãn $n=15 m^2$ với $m$ là số nguyên dương.

(b) Cho $p, q$ là các số nguyên dương và $d=5(4 p+3) q^2$. Chứng minh rằng $\Delta_d \geq 5$.

Bài 6. Với các số nguyên $a, b, c, d$ thoả mãn $1 \leq a<b<c<d$, ký hiệu:

$T(a, b, c, d)=[(x, y, z, t) \subset \mathbb{N}^* \mid 1 \leq x<y<z<t, x \leq a, y \leq b, z \leq c, t \leq d]$

(a) Tình số phần tử của $T(1,4,6,7)$.

(b) Cho $a=1$ và $b \geq 4$. Gọi $d_1$ là số phần tử của $T(a, b, c, d)$ chứa 1 và không chứa $2 ; d_2$ là số phần tử chứa 1,2 và không chứa $3 ; d_3$ là số phần tử chứa $1,2,3$ và không chứa 4 . Chứng minh rằng $d_1 \geq 2 d_2-d_3$. Đẳng thức xảy ra khi nào?

Bài 7. Trong một hệ thống máy tính, một máy tính bất kỳ có kết nối trực tiếp với ít nhất $30 \%$ máy tính khác của hệ thống. Hệ thống này có một chương trình cảnh báo và ngăn chặn khá tốt, do đó khi một máy tính bị virus, nó chỉ có đủ thời gian lây cho các máy tính được kết nối trực tiếp với nó. Chứng minh rằng dù vậy, kẻ tấn công vẫn có thể chọn hai máy tính của hệ thống mà nếu thả virus vào hai máy đó, ít nhất $50 \%$ máy tính của hệ thống sẽ bị nhiễm virus.

Bài 8. Cho tam giác $A B C$ nhọn. Đường tròn $(I)$ có tâm $I$ thuộc cạnh $B C$ và tiếp xúc với các cạnh $A B, A C$ lần lượt tại $E, F$. Lấy $M, N$ bên trong tứ giác $B C E F$ sao cho $E F N M$ nội tiếp $(I)$ và các đường thẳng $M N, E F, B C$ dồng quy. Gọi $M F$ cắt $N E$ tại $P, A P$ cắt $B C$ tại $D$.

(a) Chứng minh rằng $A, D, E, F$ cùng thuộc một đường tròn.

(b) Lấy trên các đường thẳng $B N, C M$ các điểm $H, K$ sao cho $\angle A C H=$ $\angle A B K=90^{\circ}$. Gọi $T$ là trung điểm $H K$. Chứng minh rằng $T B=T C$.

 

LỜI GIẢI

Ngày thi thứ nhất

Bài 1. Tìm tất cả $a$ để dãy số $\left(u_n\right)$ hội tụ, biết $u_1=a$ và $\forall n \in \mathbb{N}^*$ thì:

$\quad\quad\quad\quad\quad\quad\quad\quad\quad\quad u_{n+1}=\left\{\begin{array}{l}2 u_n-1 \text { nếu } u_n>0, \\ -1 \text { nếu }-1 \leq u_n \leq 0, \\ u_n^2+4 u_n+2 \text { nếu } u_n<-1\end{array}\right.$

Lời giải. Có các trường hợp sau cần xem xét:

  • Nếu $a>1$, bằng quy nạp đơn giản, ta có $u_n>1 \forall n \in \mathbb{N}^*$ và

$\quad\quad\quad\quad\quad\quad\quad\quad\quad\quad u_n=2^{n-1}(a-1)+1, \forall n \in \mathbb{N}^* .$

Do $a>1$, cho $n \rightarrow+\infty$ thì $u_n \rightarrow+\infty$. Từ đó $\left(u_n\right)$ không hội tụ.

  • Nếu $a=1$ thì $u_n=1 \forall n \in \mathbb{N}^*$ hay $\left(u_n\right)$ hội tụ về 1 .

  • Nếu $0<a<1$, ta sẽ chứng minh rằng $\left(u_n\right)$ có ít nhất một số hạng không dương. Thật vậy, giả sử $u_n>0 \forall n \in \mathbb{N}^*$ thì theo trường hợp đầu tiên, ta có:

$\quad\quad\quad\quad\quad\quad\quad\quad\quad\quad u_n=2^{n-1}(a-1)+1 \forall n \in \mathbb{N}^*$

Do $a>1$, cho $n \rightarrow+\infty$ thì $u_n \rightarrow-\infty$, trái với việc $u_n>0 \forall n, \in \mathbb{N}^*$.

Từ đó điều giả sử là sai hay phải tồn tại $k \in \mathbb{N}^*\text { sao cho } u_k>0 \text { và } u_{k+1} \leq 0$. Với cách chọn chỉ số $\text{k}$ như vậy, ta có:

$\quad\quad\quad\quad\quad\quad\quad\quad\quad\quad -1 \leq 2 u_k-1=u_{k+1} \leq 0$

Khi đó $u_{k+2}=0$. Bằng quy nạp thì $u_n=-1 \forall n \in \mathbb{N}^*, n \geq k+2$. Điều này dễn đến $\left(u_n\right)$ hội tụ về $-1$.

  • Nếu $-1 \leq a \leq 0$, từ giả thiết thì $u_2=-1$. Bằng quy nạp thì $u_n=-1 \forall n \in$ $\mathbb{N}^*, n \geq 2$ hay $\left(u_n\right)$ hội tụ về $-1$.

  • Nếu $-2<a<-1$, ta có:

$\quad\quad\quad\quad\quad\quad\quad\quad\quad\quad u_2-u_1=a^2+3 a+2=(a+2)(a+1)<0$

Khi đó thì $u_2<u_1<-1$. Lại có $u_2=(a+2)^2-2 \geq-2$ nên $-2<u_2<-1$.

Bằng quy nạp, ta có $\left(u_n\right)$ là dãy giảm và $-2<u_n<-1$ nên $\left(u_n\right)$ hội tụ.

  • Nếu $-2-\sqrt{3} \leq a \leq-2$ thì $u_2=a^2-4 a+2$ và dễ có được:

$\quad\quad\quad\quad\quad\quad\quad\quad\quad\quad\quad\quad -1 \leq a^2-4 a+2 \leq 1$

Theo các trường hợp đã xét, dãy số $\left(u_n\right)$ hội tụ.

  • Nếu $a<-2-\sqrt{3}$, bằng vài tính toán, ta có $u^2=a^2-4 a+2>1$.

Theo trường hợp đầu tiên, dãy số $\left(u_n\right)$ không hội tụ.

Vậy dãy số $\left(u_n\right)$ hội tụ khi và chỉ khi $-2-\sqrt{3} \leq a \leq 1$.

Bài 2. Tìm số nguyên dương $k$ nhỏ nhất để bất đẳng thức

$\quad\quad\quad\quad\quad\quad\quad\quad\quad\quad x^k y^k z^k\left(x^3+y^3+z^3\right) \leq 3$

luôn đúng với mọi số thực dương $x, y, z$ thoả mãn điều kiện $x+y+z=3$.

Lời giải. Ta sẽ chứng minh rằng $k=3$ là số nguyên dương nhỏ nhất thoả mãn bài toán. Trước hết, chọn $x=y=\frac{3}{4}, z=\frac{3}{2}$ thì ta phải có:

$\quad\quad\quad\quad\quad\quad\quad\quad \left(\frac{3}{4}\right)^{2 k} \cdot\left(\frac{3}{2}\right)^k\left(2 \cdot\left(\frac{3}{4}\right)^3+\left(\frac{3}{2}\right)^3\right) \leq 3$

Dễ thấy đánh giá trên chỉ đúng nếu $k \geq 3$. Ta đưa về chứng minh rằng:

$\quad\quad\quad\quad\quad\quad\quad\quad\quad\quad x^3 y^3 z^3\left(x^3+y^3+z^3\right) \leq 3 .$

Không mất tính tổng quát, giả sử $x \geq y \geq z$ thì $z \leq 1$. Ta có:

$\quad\quad\quad\quad\quad x^3+y^3=(x+y)^3-3 x y(x+y)=(3-z)^3-3 x y(x+y) \text { hay } $

$\quad\quad\quad\quad\quad (3-z)^3+z^3 \leq \frac{3}{x^3 y^3 z^3}+3 x y(x+y)$

Khai triển và thu gọn, bất đẳng thức trở thành:

$\quad\quad\quad\quad\quad\quad\quad\quad\quad\quad 3 z^2-9 z+9 \leq \frac{1}{x^3 y^3 z^3}+x^2 y+x y^2$

Theo bất đẳng thức AM-GM, ta có vế phải của bất đẳng thức trên sẽ không nhỏ hơn $\frac{3}{z}$. Từ đây ta chỉ cần chứng minh rằng

$\quad\quad\quad\quad\quad\quad\quad 3 z^2-9 z+9 \leq \frac{3}{z} \text { hay } 3(z-1)^3 \leq 0 \text {, đúng. }$

Vậy $k=3$ là hằng số nguyên dương nhỏ nhất thoả mãn bài toán.

Nhận xét. Dưới đây là các cách xử lý khác cho bất đẳng thức ứng với $k=3$ ở trên.

Cách 1. Không mất tính tổng quát ta giả sử $x \leq y \leq z$. Khi đó luôn tồn tại $m>n \geq 0$ sao cho $x=m-n, y=m+n$. Khi đó

$\quad\quad\quad\quad\quad\quad\quad\quad\quad\quad z=3-2 m ; m=\frac{x+y}{2} \leq 1$

Xét hàm số

$f(n)=(m-n)^3(m+n)^3 z^3\left[z^3+(m-n)^3+(m+n)^3\right]=z^3\left(m^2-n^2\right)^3\left(z^3+2 m^3+6 m n^2\right)$

thì

$\quad\quad\quad\quad\quad f^{\prime}(n)=z^3\left(m^2-n^2\right)^2\left(-6 n z^3-48 m n^3\right) \leq 0$

nên

$\quad\quad\quad\quad\quad f(n) \leq f(0)=m^6 z^3\left(z^3+2 m^3\right)=m^6(3-2 m)^3\left((3-2 m)^3+2 m^3\right)$

Xét hàm số

$\quad\quad\quad\quad\quad\quad\quad\quad\quad\quad g(m)=m^6(3-2 m)^3\left[(3-2 m)^3+2 m^3\right]$

thì

$\quad\quad\quad\quad\quad g^{\prime}(m)=18 m^5(3-2 m)^2(m-1)\left[(m-1)\left(8 m^2-37 m+26\right)-1\right] \geq 0 .$

Vậy nên $g(m) \leq g(1)=3$, bài toán được giải quyết.

Cách 2. Không mất tính tổng quát, ta giả sử $z$ là số lớn nhất trong ba số $x, y, z$. Đặt $t=\frac{x+y}{2}$ và $f(x, y, z)=x^3 y^3 z^3\left(x^3+y^3+z^3\right)$. Ta sẽ chứng minh $f(x, y, z) \leq$ $f(t, t, z)$. Ta có

$\quad\quad\quad\quad f(t, t, z)-f(x, y, z)=z^3\left[t^6\left(2 t^3+z^3\right)-x^3 y^3\left(x^3+y^3+z^3\right)\right] .$

Mặt khác,

$t^6\left(2 t^3+z^3\right)-x^3 y^3\left(x^3+y^3+z^3\right)=z^3\left(t^6-x^3 y^3\right)+2 t^9-x^3 y^3(x+y)\left(x^2+y^2-x y\right) $

$=z^3\left(t^6-x^3 y^3\right)+2 t^9-2 t x^3 y^3\left(4 t^2-3 x y\right) \geq t^3\left(t^6-x^3 y^3\right)+2 t^9-2 t x^3 y^3\left(4 t^2-3 x y\right) $

$=3 t\left(t^2-x y\right)\left[t^6+x y\left(2 x y+t^2\right)\left(t^2-x y\right)\right] \geq 0 .$

Vậy nên

$\quad\quad\quad\quad\quad f(x, y, z) \leq f(t, t, z)=f(t, t, 3-2 t)=t^6(3-2 t)^3\left[2 t^3+(3-2 t)^3\right]$

Ta chỉ cần chứng minh

$\quad\quad\quad\quad\quad\quad\quad\quad\quad\quad t^6(3-2 t)^3\left[2 t^3+(3-2 t)^3\right] \leq 3$

Đến đây thực hiện như cách 1 ở trên.

Bài 3. Cho hàm số $f: \mathbb{N}^* \rightarrow \mathbb{N}^*$ thoả mãn hai điều kiện sau:

i) $f$ là hàm tăng thật sự trên $\mathbb{N}^*$.

ii) $f(2 n)=2 f(n) \forall n \in \mathbb{N}^*$.

(a) Giả sử $f(1)=3$ và $p>3$ là số nguyên tố. Chứng minh rằng tồn tại số nguyên dương $n$ sao cho $f(n)$ chia hết cho $p$.

(b) Cho $q$ là số nguyên tố lẻ. Hãy xây dựng một hàm $f$ thoả mãn các điều kiện của bài toán mà $f(n)$ không chia hết cho $q$ với mọi $n$ nguyên dương.

Lời giải. (a) Đặt $A=[f(n+1)-f(n) \mid n \in \mathbb{N}^*].$

Vì $\text { f là hàm số tăng thực sự trên } \mathbb{N}^* \text { nên } A \subset \mathbb{N}^*$.

Khi đó phải tồn tại $k=\min A \text { và tồn tại } n \in \mathbb{N}^* \text { để } k=f(n+1)-f(n)$. Khi đó:

$\quad\quad\quad\quad\quad\quad\quad\quad\quad\quad f(2 n+2)-f(2 n)=2 f(n+1)-2 f(n)=2 k .$

Lại có $f(2 n+2)-f(2 n+1), f(2 n+1)-f(2 n) \geq k$ nên

$\quad\quad\quad\quad\quad\quad\quad\quad\quad\quad f(2 n+2)-f(2 n+1)+f(2 n+1)-f(2 n) \geq 2 k .$

Từ đây ta phải có $f(2 n+2)-f(2 n+1)=f(2 n+1)-f(2 n)=k$. Bằng quy nạp theo $m$, ta chứng minh được

$\quad\quad\quad\quad\quad\quad\quad\quad\quad\quad f\left(2^m n+t\right)=2^m f(n)+t k \forall t, m \in \mathbb{N}, t \leq m .$

Lại có $f(1)=3, f(2)=6$ nên $k \leq 3<p$ hay $(k, p)=1$.

Xét $p$ số nguyên dương sau:

$\quad\quad\quad\quad\quad\quad\quad\quad\quad\quad f\left(2^p n\right), f\left(2^p n+1\right), f\left(2^p n+2\right), \ldots, f\left(2^p n+p-1\right)$

lập thành một cấp số cộng có công sai $k$ nên là một hệ thặng dư đầy đủ modulo $p$. Từ đó phải tồn tại một số hạng chia hết cho $p$.

(b) Ta xây dựng một hàm số $f$ với các điều kiện như sau:

$\quad\quad$ i) $f(1)=2^a>q\left(a \in \mathbb{N}^*\right.$,

$\quad\quad$ ii) $f(2 n)=2 f(n) \forall n \in \mathbb{N}^*$,

$\quad\quad$ iii) $f(2 n+1)=f(2 n)+q \forall n \in \mathbb{N}^*$.

Ta chứng minh rằng hàm số $f$ vừa xây dựng thỏa mãn bài toán.

Trước hết ta chứng minh rằng $f$ là hàm tăng thực sự, cụ thể là:

$\quad\quad\quad\quad\quad\quad\quad\quad\quad\quad f(n+1)-f(n) \geq q \forall n \in \mathbb{N}^* .$

Với $n=1$, ta có $f(2)-f(1)=2.2^a-2^a=2^a>q$. Giả sử khẳng định cần chứng minh đúng đến $n=k$. Xét các khả năng sau:

  • Nếu $k$ là số chẵn, ta có $f(k+1)=f(k)+q$ thỏa mãn yêu cầu.

  • Nếu $k$ là số lẻ, ta có:

$\quad\quad\quad\quad f(k+1)=2 f\left(\frac{k+1}{2}\right) \geq 2\left(f\left(\frac{k-1}{2}\right)+q\right)=f(k-1)+2 q .$

Lại có $f(k)=f(k-1)+q$ nên $f(k+1) \geq f(k)+q$.

Theo nguyên lý quy nạp, ta có $f(n+1)-f(n) \geq q \forall n \in \mathbb{N}^*$.

Bây giờ ta chứng minh rằng không tồn tại $n$ để $q \mid f(n)$. Trước hết thì $f(1)=2^a$ không chia hết cho $q$. Giả sử điều này đúng đến $n=k$. Xét các khả năng sau:

  • Nếu $k$ chẵn thì $f(k+1)=f(k)+q$ không chia hết cho $q$.

  • Nếu $k$ lẻ thì $f(k+1)=2 f\left(\frac{k+1}{2}\right)$ không chia hết cho $q$.

Theo nguyên lý quy nạp, $f(n)$ không chia hết cho $q$ với mọi $n \in \mathbb{N}^*$. Các điều kiện đã được kiểm tra đầy đủ.

Bài 4. Cho tam giác $A B C$ có góc $\angle B A C$ tù và $A H \perp B C(H$ nằm trên $B C$ ). Điểm $M$ thay đổi trên cạnh $A B$. Dựng điểm $N$ sao cho $\Delta B M N \sim \triangle H C A$, với $H$ và $N$ nằm khác phía đối với đường thẳng $A B$.

(a) Gọi $C M$ cắt đường tròn ngoại tiếp tam giác $B M N$ tại $K$. Chứng minh rằng $N K$ luôn đi qua một điểm cố định.

(b) Gọi $N H$ cắt $A C$ tại $P$. Dựng điểm $Q$ sao cho $\triangle H P Q \sim \Delta H N M$, với $Q$ và $M$ nằm khác phía đối với đường thẳng $N P$. Chứng minh rằng $Q$ luôn thuộc một đường thẳng cố định.

Lời giải. (a) Xét điểm $X$ trên $A C$ sao cho $\angle X B C=90^{\circ}$ và $K^{\prime}$ là giao điểm của $N X$ và $C M$. Ta có $\Delta B M N \sim \triangle B C X$ (cùng hướng). Từ đó có một phép vị tự quay tâm $B$ biến $M \mapsto N, C \mapsto X$.

Giả sử $C M$ cắt $B X$ tại $K^{\prime}$ thì $K^{\prime}$ thuộc đường tròn ngoại tiếp tam giác $B M N$. Từ đó $K^{\prime} \equiv K$ nên $N K$ luôn đi qua điểm $X$ cố định.

(b) Xét phép vị tự tâm $H$ biến

$\quad\quad\quad\quad\quad\quad\quad\quad\quad\quad N \mapsto P, M \mapsto Q, B \mapsto F .$

Ta có $\Delta B M N \sim \triangle F Q P$. Khi đó

$\quad\quad\quad\quad\quad\quad\quad\quad\quad\quad \angle F Q P=\angle B M N=\angle A C B=\angle F C P$

nên tứ giác $C F P Q$ nội tiếp. Từ đây dẫn đến

$\quad\quad\quad\quad\quad\quad\quad\quad\quad\quad \angle Q C P=\angle Q F P=\angle M B N=90^{\circ} .$

Vậy $Q$ thuộc đường thẳng qua $C$ vuông góc với $A C$, là đường thẳng cố định.

 

Ngày thi thứ hai

Bài 5. Với mỗi số nguyên dương $n$, tồn tại duy nhất số tự nhiên $a$ thoả mãn điều kiện $a^2 \leq n<(a+1)^2$. Đặt $\Delta_n=n-a^2$.

(a) Tìm giá trị nhỏ nhất của $\Delta_n$ khi $n$ thay đổi và luôn thoả mãn $n=15 m^2$ với $m$ là số nguyên dương.

(b) Cho $p, q$ là các số nguyên dương và $d=5(4 p+3) q^2$. Chứng minh rằng $\Delta_d \geq 5$.

Lời giải. (a) Ta cần tìm $\Delta_n$ nhỏ nhất để phương trình $15 m^2-a^2=\Delta_n$ có nghiệm nguyên dương. Nhận thấy $15-3^2=6$ nên $\min \Delta_n \leq 6$. Ta chứng minh rằng phương trình trên không có nghiệm nguyên dương với $\Delta_n<6$.

Ta có $3 \mid a^2+\Delta_n$. Suy ra $3 \mid \Delta_n$ hoặc $3 \mid \Delta_n+1$. Mặt khác $5 \mid a^2+\Delta_n$ nên $\Delta_n$ chia 5 chỉ có thể dư 0,1 hoặc 4 .

Từ đó nếu tồn tại $n$ để $\Delta_n<6$ thỏa mãn bài toán thì $\Delta_n=5$. Giả sử rằng tồn tại $n$ như thế, ta có $15 m^2-a^2=5$ hay $5 \mid a$. Đặt $a=5 s\left(s \in \mathbb{N}^*\right)$, ta có:

$\quad\quad\quad\quad\quad\quad\quad\quad\quad\quad\quad\quad 3 m^2-5 s^2=1 \text {. }$

Từ đó thì

$\quad\quad\quad\quad 3\left(m^2+s^2\right) \equiv 1 \quad(\bmod 8)$ hay $m^2+s^2 \equiv 3 \quad(\bmod 8)$

Điều này vô lý do $m^2$ chia 8 dư $0,1,4$. Vậy $\Delta_n$ nhỏ nhất là 6 .

(b) Ta có

$\quad\quad\quad\quad\quad\quad\quad\quad\quad\quad 5(4 p+3) q^2-a^2=\Delta_d .$

Do $a^2$ chia 5 dư $0,1,4$ nên $\Delta_d$ chia 5 dư $0,1,4$. Giả sử rằng có bộ số để $\Delta_d<5$. Xét các khả năng sau:

  • Nếu $\Delta_d=0$ thì $5(4 p+3) q^2=a^2$. Xét bộ số $(q, a)$ với $q+a$ nhỏ nhất. Từ phương trình trên, ta có $a^2+q^2 \equiv 0(\bmod 4)$ hay $a \equiv q \equiv 0(\bmod 2)$.

Đặt $a=2 a_1$ và $q=2 q_1$ với $a_1, q_1 \in \mathbb{N}^*$ thì bộ số $\left(q_1, a_1\right)$ cũng thoả mãn điều kiện $5(4 p+3) q_1^2=a_1^2$. Hơn nữa $q_1+a_1<q+a$, mâu thuẫn.

  • Nếu $\Delta_d=1$, ta có $a^2+1=5(4 p+3) q^2$. Do $5(4 p+3) \equiv 3(\bmod 4)$ nên số này tồn tại một ước nguyên tố $r \equiv 3(\bmod 4)$.

Do đó $a^2+1 \equiv 0(\bmod r)$ hay $r \mid 1$, vô lý.

  • Nếu $\Delta_d=4$, chứng minh tương tự, ta cũng có điều mâu thuẫn.

Vậy ta phải có $\Delta_d \geq 5$.

Bài 6. Với các số nguyên $a, b, c, d$ thoả mãn $1 \leq a<b<c<d$, ký hiệu: $T(a, b, c, d)=[(x, y, z, t) \subset \mathbb{N}^* \mid 1 \leq x<y<z<t, x \leq a, y \leq b, z \leq c, t \leq d]$.

(a) Tính số phần tử của $T(1,4,6,7)$.

(b) Cho $a=1$ và $b \geq 4$. Gọi $d_1$ là số phần tử của $T(a, b, c, d)$ chứa 1 và không chứa $2 ; d_2$ là số phần tử chứa 1,2 và không chứa $3 ; d_3$ là số phần tử chứa $1,2,3$ và không chứa 4 . Chứng minh rằng $d_1 \geq 2 d_2-d_3$. Đẳng thức xảy ra khi nào ?

Lời giải. (a) Với $T(1,4,6,7)$, ta có $x \leq 1$ nên $x=1$. Khi đó ta có $2 \leq y \leq 4$ hay $y \in{2,3,4}$. Xét các khả năng sau:

  • Nếu $y=2$ thì $3 \leq z \leq 6$. Với mỗi giá trị của $z$, ta có thể thu được $7-z$ giá trị của $t$ nên ta có 10 bộ số.

  • Nếu $y=3$, tương tự ta có 6 bộ số.

  • Nếu $y=4$, tương tự ta có 3 bộ số.

Vậy có tất cả 19 bộ số trong $T(1,4,6,7)$.

(b) Đặt các tập hợp sau:

$\quad\quad\quad\quad\quad\quad\quad \left\{\begin{array}{l}T_1={(1, y, z, t) \mid 3 \leq y \leq b, y<z \leq c, z<t \leq d} \\ T_2={(1,2, z, t) \mid 4 \leq z \leq c, z<t \leq d} \\ T_3={(1,2,3, t) \mid 5 \leq t \leq d}\end{array}\right.$

Ta có $d_3=\left|T_3\right|=d-4$ và

$\quad\quad\quad\quad\quad\quad\quad\quad\quad\quad d_2=\sum_{z=4}^c(d-z)=(c-3) d+\frac{(c+4)(c-3)}{2}$

Tiếp theo ta tính $d_1=\left|T_1\right|$. Vì $b \geq 4$ nên $y \geq 3$. Xét các khả năng sau

  • Nếu $y=3$ thì $T(1,3, z, t)=d_2$.

  • Nếu $y=4$ thì $T(1,4, z, t)=\sum_{z=5}^c(d-z)=(c-4) d-\frac{(c+5)(c-4)}{2}$.

Từ đó $d_1 \geq d_2+(c-4) d-\frac{(c+5)(c-4)}{2}$. Do đó, kết hợp với việc tính được giá trị của $d_2$, khi cộng theo vế thì $d_1+d_3-2 d_2 \geq 0$.

Vậy $d_1 \geq 2 d_2-d_3$. Đẳng thức xảy ra khi và chỉ khi $b=4$.

Nhận xét. Ngoài lời giải khá “đại số” phía trên, có một lời giải khác cho ý sau của bài toán sử dụng song ánh:

  • Điểm mấu chốt là phân rã $T_1, T_2, T_3$ thành các nhóm thích hợp và thiết lập được đơn ánh giữa chúng. Với các tập $T_1, T_2, T_3$ định nghĩa như trên, ta viết $T_1$ thành $A \cup B \cup C$ có giao đôi một khác rỗng, trong đó

$\quad\quad\quad\quad\quad\quad \left\{\begin{array}{l}A={(1,3,4, t) \mid 5 \leq t \leq d} \\ B={(1,3, z, t) \mid 5 \leq z \leq c, z<t \leq d} \\ C={(1, y, z, t) \mid 4 \leq y \leq b, y<z \leq c, z<t \leq d}\end{array}\right.$

  • Dễ kiểm chứng rằng có song ánh từ $A$ vào $T_3$ nên $|A|=\left|T_3\right|=d_3$.

  • Xét $D={(1,4, z, t) \mid 5 \leq z \leq c, z<t \leq d}$. Dễ kiểm chứng rằng $D \subset C$ và có song ánh từ $D$ vào $B$ nên $|D|=|B|$.

  • Ta có $A \cup B={(1,3, z, t) \mid 4 \leq z \leq c, z<t \leq d}$. Dễ kiểm chứng rằng có song ánh từ $A \cup B$ vào $T_2$ nên $|A \cup B|=\left|T_2\right|=d_2$. Chú ý rằng $A \cap B=\varnothing$ nên $|A|+|B|=d_2$ hay $|B|=d_2-d_3$. Từ đó ta có:

$\quad\quad\quad\quad\quad d_1=|A|+|B|+|C| \geq|A|+|B|+|D|=d_3+2|B|$

Vậy $d_1 \geq d_3+2\left(d_2-d_3\right)=2 d_2-d_3$. Đẳng thức xảy ra khi và chỉ khi $b=4$.

Bài 7. Trong một hệ thống máy tính, một máy tính bất kỳ có kết nối trực tiếp với ít nhất $30 \%$ máy tính khác của hệ thống. Hệ thống này có một chương trình cảnh báo và ngăn chặn khá tốt, do đó khi một máy tính bị virus, nó chỉ có đủ thời gian lây cho các máy tính được kết nối trực tiếp với nó. Chứng minh rằng dù vậy, kẻ tấn công vẫn có thể chọn hai máy tính của hệ thống mà nếu thả virus vào hai máy đó, ít nhất $50 \%$ máy tính của hệ thống sẽ bị nhiễm virus.

Lời giải Trước hết ta chứng minh bổ đề sau: Xét một tập con $S$ bất kỳ của tập các máy tính $X$, khi đó tồn tại 1 máy tính của hệ thống kết nối trực tiếp với ít nhất $30 \%$ máy tính của $S$.

Thật vậy, xét các cặp $(s, x)$ với $s \in S, x \in X$ và $(s, x)$ kết nối trực tiếp với nhau. Khi đó, nếu tính theo $s$ thì số cặp như vậy sẽ không ít hơn $\frac{3}{10}|S||X|$. Do đó nếu tính theo $x$ thì sẽ phải tồn tại máy tính $x$ kết nối trực tiếp với ít nhất $\frac{3}{10}|S|$.

Quay trở lại bài toán,

Giả sử hệ thống có $n$ máy tính. Xét máy tính $A$ bất kỳ. Gọi $S$ là tập hợp các máy tính không kết nối trực tiếp với $A$. Nếu $S=\varnothing$ thì kết quả bài toán là hiển nhiên. Nếu $S \neq \varnothing$ thì theo bổ đề, tồn tại máy tính $B$ kết nối trực tiếp với ít nhất $30 \%$ máy tính trong $S$. Ta chứng minh hai máy tính $A$ và $B$ thỏa mãn yêu cầu bài toán.

Thật vậy, giả sử $A$ kết nối trực tiếp với $k$ máy tính khác. Khi đó, theo cách chọn, $A$ và $B$ sẽ kết nối trực tiếp với ít nhất

$\quad\quad\quad\quad\quad k+0,3(n-k)=0,7 k+0,3 n \geq 0,7 \cdot 0,3 n+0,3 n=0,51 n .$

Từ đây ta có được kết luận của bài toán.

Bài 8 . Cho tam giác $A B C$ nhọn. Đường tròn $(I)$ có tâm $I$ thuộc cạnh $B C$ và tiếp xúc với các cạnh $A B, A C$ lần lượt tại $E, F$. Lấy $M, N$ bên trong tứ giác $B C E F$ sao cho $E F N M$ nội tiếp $(I)$ và các đường thẳng $M N, E F, B C$ đồng quy. Gọi $M F$ cắt $N E$ tại $P, A P$ cắt $B C$ tại $D$.

(a) Chứng minh rằng $A, D, E, F$ cùng thuộc một đường tròn.

(b) Lấy trên các đường thẳng $B N, C M$ các điểm $H, K$ sao cho $\angle A C H=$ $\angle A B K=90^{\circ}$. Gọi $T$ là trung điểm $H K$. Chứng minh rằng $T B=T C$.

Lời giải. (a) Ta sẽ chứng minh rằng $A D \perp B C$. Gọi $X$ là điểm đồng quy của $E F, M N, B C$. Do $A E, A F$ tiếp xúc với $(I)$ nên $E F$ là đường đối cực của $A$ đối với (I). Ta có $X \in E F$ nên theo định lý La Hire, điểm $A$ sẽ nằm trên đường đối cực của $X$ đối với đường tròn $(I)$.

Lại có $P$ là giao điểm của $E N, F M$ nên $P$ nằm trên đường đối cực của $X$ đối với $(I)$. Vì thế nên $A P$ là đường đối cực của $X$ đối với $(I)$ hay $A P \perp B C$. Do đó

$\quad\quad\quad\quad\quad\quad\quad\quad\quad\quad \angle A D I=\angle A E I=\angle A F I=90^{\circ} .$

Vậy $A, D, E, F$ cùng thuộc một đường tròn.

(b) Gọi $S$ là giao điểm của $B N, C M$. Xét hai tam giác $P E F, S B C$ có $P E$ cắt $S B$ tại $N, P F$ cắt $S C$ tại $M, E F$ cắt $B C$ tại $X$ và $X, M, N$ thẳng hàng. Theo định lý Desargues thì $P S, E B, F C$ đồng quy. Mặt khác $E B$ cắt $F C$ tại $A$ nên $A, P, S$ thẳng hàng, dẫn đến $S \in A D$.

Tiếp theo ta sẽ chứng minh rằng $\angle B A K=\angle C A H$. Áp dụng định lý Ceva dạng lượng giác cho tam giác $A B C$ với:

  • Các đường thẳng $A D, B H, C K$ đồng quy:

$\quad\quad\quad\quad\quad\quad\quad\quad\quad\quad \frac{\sin \angle D A B}{\sin \angle D A C} \cdot \frac{\sin \angle H B C}{\sin \angle H B A} \cdot \frac{\sin \angle K C A}{\sin \angle K C B}=1$

  • Các đường thẳng $A H, B H, C H$ đồng quy:

$\quad\quad\quad\quad\quad\quad\quad\quad\quad\quad \frac{\sin \angle H A B}{\sin \angle H A C} \cdot \frac{\sin \angle H B C}{\sin \angle H B A} \cdot \frac{\sin \angle H C A}{\sin \angle H C B}=1$

  • Các đường thẳng $A K, B K, C K$ đồng quy:

$\quad\quad\quad\quad\quad\quad\quad\quad\quad\quad \frac{\sin \angle K A B}{\sin \angle K A C} \cdot \frac{\sin \angle K B C}{\sin \angle K B A} \cdot \frac{\sin \angle K C A}{\sin \angle K C B}=1$

Chú ý rằng do các góc vuông và góc bù nhau nên ta có

$\quad\quad\quad\quad\quad\quad\quad\quad\quad\quad \frac{\sin \angle H A C}{\sin \angle H A B}=\frac{\sin \angle K A B}{\sin \angle K A C}$

Từ đó sử dụng công thức cộng cho mẫu thức và biến đổi thì:

$\quad\quad\quad\quad\quad\quad\quad\quad\quad\quad \tan \angle H A C=\tan \angle K A B$

Dẫn đến $\angle H A C=\angle K A B$. Cuối cùng, ta sẽ chứng minh $T B=T C$.

Gọi $U, V$ lần lượt là trung điểm của các đoạn $A K, A H$. Ta có:

$\quad\quad\quad\quad\quad\quad\quad\quad\quad\quad U B=\frac{A K}{2}=V T, U T=\frac{A H}{2}=V C .$

Đồng thời, ta cũng có:

$\quad\quad\quad\quad\quad\quad \angle B U T=\angle B U A-\angle A U T=\angle A V C-\angle A V T=\angle T V C$

Do đó $\Delta B U T=\Delta T V C$ (c.g.c), vậy nên $T B=T C$.

Nhận xét. Để chứng minh $\angle H A C=\angle K A B$, cũng là mấu chốt của lời giải trên, ta có thể dùng bổ đề sau:

Cho tam giác $A B C$ có hai điểm $P, Q$ sao cho $A P, A Q$ đẳng giác trong góc $A$. Gọi $X$ là giao điểm của $B P, C Q$ và $Y$ là giao điểm của $B Q, C P$. Khi đó, ta cũng có $A X, A Y$ đẳng giác trong góc $A$.

 

 

 

 

 

 

 

 

 

 

 

 

 

 

 

 

 

 

 

 

 

 

 

 

 

 

 

 

 

 

 

 

 

 

 

 

 

 

 

 

 

 

 

 

 

 

 

 

 

 

 

 

 

 

 

 

 

 

 

 

 

ĐỀ THI VÀO LỚP 10 CHUYÊN TOÁN TP.HCM NĂM 2016

Bài 1. (a) Cho hai số thực $a, b$ sao cho $|a| \neq|b|$ và $a b \neq 0$ thỏa mãn điều kiện: $\frac{a-b}{a^{2}+a b}+\frac{a+b}{a^{2}-a b}=\frac{3 a-b}{a^{2}-b^{2}}$. Tính giá trị biểu thức: $P=\frac{a^{3}+2 a^{2} b+3 b^{3}}{2 a^{3}+a b^{2}+b^{3}}$

(b) Cho $m, n$ là các số nguyên dương sao cho $5 m+n$ chia hết cho $5 n+m$. Chứng minh rằng: $m \vdots n$.

Bài 2. (a) Giải phương trình: $x^{2}-6 x+4+2 \sqrt{2 x-1}=0$

(b) Giải hệ phương trình: $\left\{\begin{array}{l}x^{3}-y^{3}=9(x+y) \\ x^{2}-y^{2}=3\end{array}\right.$

Bài 3. Cho tam giác nhọn $\triangle A B C$ có các đường cao $A A_{1}, B B_{1}, C C_{1}$. Gọi $K$ là hình chiếu của $A$ lên $A_{1} B_{1} ; L$ là hình chiếu của $B$ lên $B_{1} C_{1}$. Chứng minh rằng $A_{1} K=B_{1} L$.

Bài 4. Cho $x, y$ là hai số thực dương. Chứng minh rằng:

$\frac{x \sqrt{y}+y \sqrt{x}}{x+y}-\frac{x+y}{2} \leq \frac{1}{4}$

Bài 5. Cho tứ giác nội tiếp $A B C D$ có $A C$ cắt $B D$ tại $E$. Tia $A D$ cắt tia $B C$ tại $F$. Dựng hình bình hành $A E B G$.

(a) Chứng minh rằng: $F D \cdot F G=F B . F E$

(b) Gọi $H$ là điểm đối xứng của $E$ qua $A D$. Chứng minh rằng 4 điểm $F, H, A, G$ cùng thuộc một đường tròn.

Bài 6. Nam cắt một tờ giấy ra làm 4 miếng hoặc 8 miếng rồi lấy một số miếng nhỏ đó cắt ra làm 4 miếng hoặc 8 miếng nhỏ hơn và Nam cứ tiếp tục thực hiện việc cắt như thế nhiều lần. Hỏi với việc cắt này, Nam có thể cắt được thành 2016 miếng lớn, nhỏ hay không? Vi sao?

LỜI GIẢI

Bài 1.

a) Cho hai số thực $a, b$ sao cho $|a| \neq|b|$ và $a b \neq 0$ thỏa mãn điều kiện: $\frac{a-b}{a^{2}+a b}+\frac{a+b}{a^{2}-a b}=\frac{3 a-b}{a^{2}-b^{2}}$. Tính giá trị biểu thức: $P=\frac{a^{3}+2 a^{2} b+3 b^{3}}{2 a^{3}+a b^{2}+b^{3}}$

b) Cho $m, n$ là các số nguyên dương sao cho $5 m+n$ chia hết cho $5 n+m$. Chứng minh rằng: $m \vdots n$.

Lời giải.

a) Ta có:

$\frac{a-b}{a^{2}+a b}+\frac{a+b}{a^{2}-a b}=\frac{3 a-b}{a^{2}-b^{2}}$

$\Leftrightarrow \frac{a-b}{a(a+b)}+\frac{a+b}{a(a-b)}=\frac{3 a-b}{(a+b)(a-b)}$

$\Leftrightarrow(a-b)^{2}+(a+b)^{2}=a(3 a-b) \Leftrightarrow a^{2}-a b-2 b^{2}=0$

$\Leftrightarrow(a+b)(a-2 b)=0 \Leftrightarrow a=2 b(|a| \neq|b|)$

Từ đó ta tính được $P: P=\frac{a^{3}+2 a^{2} b+3 b^{3}}{2 a^{3}+a b^{2}+b^{3}}=\frac{8 b^{3}+8 b^{3}+3 b^{3}}{16 b^{3}+2 b^{3}+b^{3}}=1$

b) Từ giả thuyêt ta có thể giả sử: $k=\frac{5 m+n}{5 n+m}$, với $k \in N^{*}$

Dễ dàng suy ra được: $\frac{m}{n}=\frac{5 k-1}{5-k}$

Do $m$ và $n$ là hai số nguyên dương nên: $\frac{m}{n}>0$

Từ đó ta có hai trường hợp sau:

Trường hợp 1: $\left\{\begin{array}{l}5 k-1>0 \\ 5-k>0\end{array} ; 5>k>\frac{1}{5}\right.$

Từ đó ta có thể tính được $k=1,2,3,4$. Thay lần lượt các giá trị của $k$ vào biểu thức ta đều thu được $m \vdots n$

Trường hợp 2: $\left\{\begin{array}{c}5 k-1<0 \\ 5-k<0\end{array} ; k>5, k<\frac{1}{5}\right.$

Trường hợp này không tồn tại giá trị của $k$ thỏa yêu cầu bài toán, từ đó ta luôn có: $m \vdots n$

Bài 2.

a) Giải phương trình: $x^{2}-6 x+4+2 \sqrt{2 x-1}=0$

b) Giải hệ phương trình: $\left\{\begin{array}{l}x^{3}-y^{3}=9(x+y) \\ x^{2}-y^{2}=3\end{array}\right.$

Lời giải.

a) Điều kiện xác định: $x \geq \frac{1}{2}$

Ta có:

$x^{2}-6 x+4+2 \sqrt{2 x-1}=0 \Leftrightarrow x^{2}-4 x+4=(2 x-1)-2 \sqrt{2 x-1}+1 $

$\Leftrightarrow(x-2)^{2}=(\sqrt{2 x-1}-1)^{2} \Leftrightarrow|x-2|=|\sqrt{2 x-1}-1|$

Trường hợp 1: $x \geq 2$. Từ phương trình đã cho ta thu được:

$x-2=\sqrt{2 x-1}-1 \Leftrightarrow x-1=\sqrt{2 x-1}$

Từ đó ta có:

$\Leftrightarrow x^{2}-2 x+1=2 x-1 \Leftrightarrow x^{2}-4 x+2=0$

Kết hợp với điều kiện $x \geq 2$ ta tìm được nghiệm duy nhất của phương trình là: $x=2+\sqrt{2}$

Trường hợp 2: $1 \leq x<2$. Từ phương trình đã cho ta thu được:

$2-x=\sqrt{2 x-1}-1 \Leftrightarrow 3-x=\sqrt{2 x-1}$

Từ đó ta có:

$\Leftrightarrow x^{2}-6 x+9=2 x-1 \Leftrightarrow x^{2}-8 x+10=0$

Kết hợp với điều kiện $1 \leq x<2$ ta nhận thấy phương trình này vô nghiệm

Trường hợp 3: $1>x \geq \frac{1}{2}$. Từ phương trình đã cho ta thu được:

$2-x=1-\sqrt{2 x-1} \Leftrightarrow x-1=\sqrt{2 x-1}$

Phương trình này vô nghiệm vì vế trái của phương trình là một số âm và vế phải của phương trình là một số không âm.

Vậy nghiệm duy nhất của phương trình là: $x=2+\sqrt{2}$

b) $\left\{\begin{array}{l}x^{3}-y^{3}=9(x+y) \\ x^{2}-y^{2}=3\end{array}\right.$

Đây là một hệ phương trình gồm các phương trình không đồng bậc, ý tưởng rất đơn giản ta sẽ chuyển nó về một phương trình đồng bậc để giải. Như vậy ta sẽ thay phương trình thứ hai vào phương trình thứ nhât. Từ đó ta thu được:

$x^{3}-y^{3}=3\left(x^{2}-y^{2}\right)(x+y)=3(x-y)(x+y)^{2} $

$\Leftrightarrow(x-y)\left(2 x^{2}+5 x y+2 y^{2}\right)=0 \Leftrightarrow(x-y)(2 x+y)(x+2 y)=0$

Từ đây ta thu được các trường hợp sau: $x=y$ hoặc $x=-2 y$ hoạcc $y=-2 x$.

Trong đó từ phương trình thứ hai ta có thể thấy $x^{2}>y^{2}$, từ đó các trường hợp $x=y$ và $y=-2 x$ bị loại. Ta chỉ có duy nhất trường hợp $x=-2 y$. Thay vào phương trình thứ hai ta lần ta có thể tính được các nghiệm của phương trình này là: $(x ; y)={(-2 ; 1),(2 ;-1)}$

Bài 3. Cho tam giác nhọn $\triangle A B C$ có các đường cao $A A_{1}, B B_{1}, C C_{1}$. Gọi $K$ là hình chiếu của $A$ lên $A_{1} B_{1}$; L là hình chiếu của $B$ lên $B_{1} C_{1}$. Chứng minh rằng $A_{1} K=B_{1} L$.

Lời giải.

Cách 1: Ta có thể suy ra điều phải chứng minh từ các cặp tam giác đồng dạng sau đây: $\triangle B_{1} B L \backsim \triangle A B A_{1}, \triangle A A_{1} K \backsim \triangle A C C_{1}, \triangle A B B_{1} \backsim \triangle A C C_{1}$ Từ đây ta có các tỷ số:

$\frac{B_{1} B}{A B}=\frac{B_{1} L}{A A_{1}}=\frac{B L}{B A_{1}} \Rightarrow \frac{B_{1} L}{B_{1} B}=\frac{A A_{1}}{A B} $

$\frac{A A_{1}}{A C}=\frac{A K}{A C_{1}}=\frac{A_{1} K}{C C_{1}} \Rightarrow \frac{A_{1} K}{C C_{1}}=\frac{A A_{1}}{A C} $

$\frac{A B}{A C}=\frac{B B_{1}}{C C_{1}}$

Từ đó hai hệ thức đầu ta có:

$\frac{B_{1} L}{A_{1} K} \cdot \frac{C C_{1}}{B B_{1}}=\frac{A C}{A B} \Rightarrow \frac{B_{1} L}{A_{1} K}=\frac{A C}{A B} \cdot \frac{B B_{1}}{C C_{1}}=1$

Vậy $A_{1} K=B_{1} L$ (đpcm)

Cách 2: Xét tam giác $\triangle A A_{1} K$, ta có: $A_{1} K=A A_{1} \sin \angle A_{1} A K$

Do $\angle A B_{1} B=\angle A A_{1} B$. Nên tứ giác $A B_{1} A_{1} B$ là tứ giác nội tiếp. Suy ra $\angle B A A_{1}=\angle B B_{1} A_{1}$

Mà: $\angle B B_{1} A_{1}+\angle A_{1} B_{1} C=90^{\circ}$ và $\angle A_{1} B_{1} C=\angle A B_{1} K$ (hai góc đối đỉnh)

Nên: $\angle B B_{1} A_{1}+\angle A B_{1} K=90^{\circ}$

Mà: $\angle A B_{1} K+\angle B_{1} A K=90^{\circ}$ (Do tam giác $A B_{1} K$ vuông tại $K$ )

Suy ra: $\angle B B_{1} A_{1}=\angle B_{1} A K=\angle B A A_{1}$

Vậy: $\angle A_{1} A K=\angle B A C$

Từ đó ta có: $A_{1} K=A A_{1} \sin \angle B A C$

Chứng minh tương tự ta được: $B_{1} L=B B_{1} \sin \angle A B C$

Vậy: $\frac{B_{1} L}{A_{1} K}=\frac{B B_{1} \sin (A B C)}{A A_{1} \sin (B A C)}$

Dễ dàng chứng minh được công thức sau đây:

$S_{A B C}=\frac{1}{2} B A \cdot B C \cdot \sin \angle A B C$

$S_{A B C}=\frac{1}{2} B A \cdot A C \cdot \sin \angle B A C$

Từ đó ta có:

$\frac{B_{1} L}{A_{1} K}=\frac{B B_{1} \sin (A B C)}{A A_{1} \sin (B A C)}=\frac{A C \cdot B B_{1}}{B C \cdot A A_{1}}=1$

Bài 4. Cho $x, y$ là hai số thực dương. Chứng minh rằng:

$\frac{x \sqrt{y}+y \sqrt{x}}{x+y}-\frac{x+y}{2} \leq \frac{1}{4}$

Lời giải. Bằng biến đổi tương đương ta thu được bất đẳng thức cần chứng minh trở thành:

$4 \sqrt{x y}(\sqrt{x}+\sqrt{y}) \leq(x+y)[2(x+y)+1]$

Vì $x>0, y>0$. Áp dụng bất đẳng thức Cauchy: $x+\frac{1}{4} \geq \sqrt{x}, y+\frac{1}{4} \geq \sqrt{y}$ Cộng vế theo vế ta thu được: $x+y+\frac{1}{2} \geq \sqrt{x}+\sqrt{y}$. Hay: $2(x+y)+1 \geq 2(\sqrt{x}+\sqrt{y})$ Lại áp dụng bất đẳng thức Cauchy một lần nữa ta có: $x+y \geq 2 \sqrt{x y}$

Nhân hai vế lại với nhau ta có:

$4 \sqrt{x y}(\sqrt{x}+\sqrt{y}) \leq(x+y)[2(x+y)+1]$

Dấu bằng của bất đẳng thức này xảy ra khi: $x=y=\frac{1}{4}$

Bài 5. Cho tứ giác nội tiếp $A B C D$ có $A C$ cắt $B D$ tại $E$. Tia $A D$ cắt tia $B C$ tại $F$. Dựng hình bình hành $A E B G$.

a) Chứng minh rằng: $F D \cdot F G=F B . F E$

b) Gọi $H$ là điểm đối xứng của $E$ qua $A D$. Chứng minh rằng 4 điểm $F, H, A, G$ cùng thuộc một đường tròn.

Lời giải.

a) Chứng minh rằng: $\frac{F B}{F D}=\frac{F G}{F E}$

Ta có: $\triangle F B A \backsim \triangle F D C$. Từ đó ta có các tỷ số:

$\frac{F B}{F D}=\frac{A B}{D C}=\frac{F A}{F C}$

Ta có: $\triangle A G B \backsim \triangle C E D$. Từ đó ta có các tỷ số:

$\frac{A B}{D C}=\frac{G B}{E D}=\frac{A G}{C E}$

Ta có: $\triangle F E D \backsim \triangle F G B$. Từ đó ta có các tỷ số:

$\frac{F E}{F G}=\frac{E D}{G B}=\frac{F D}{F B}$

Kết hợp cả ba tỷ số bằng nhau trên ta có: $\frac{F B}{F D}=\frac{F G}{F E}$

b) Chứng minh rằng: $F, H, A, G$ cùng thuộc một đường tròn Chứng minh tương tự như trên ta có: $\triangle C E F \backsim \triangle A G F$ Từ đó ta có: $\angle A G F=\angle C E F$

Mà: $\angle A H F=\angle A E F$. Do $H$ đối xứng với $E$ qua $A F$ Và: $\angle A E F+\angle C E F=180^{\circ}$. Do 3 điểm $A, C, E$ thằng hàng Vậy: $\angle A G F+\angle A H F=180^{\circ}$

Nên 4 điểm $F, H, A, G$ cùng thuộc một đường tròn

 

Bài 6. Nam cắt một tờ giấy ra làm 4 miếng hoặc 8 miếng rồi lấy một số miếng nhỏ đó cắt ra làm 4 miếng hoặc 8 miếng nhỏ hơn và Nam cứ tiếp tục thực hiện việc cắt như thế nhiều lần. Hỏi với việc cắt này, Nam có thể cắt được thành 2016 miếng lớn, nhỏ hay không? Vi sao?

Lời giải.

Gọi $x$ là số miếng giấy Nam có được sau $k$ lần cắt $\left(x ; k \in N^{*}\right)$. Vì lúc đầu Nam có 1 miếng giấy và mỗi lần cắt một miếng giấy ra làm 4 miếng hoặc làm 8 miếng nên sau mỗi lần cắt, số miếng giấy tăng thêm 3 hoặc 7 miếng, do đó ta có: $x \equiv 1(\bmod 3)$ hoặc $x \equiv 1(\bmod 7)$. Vi $2016 \equiv 0(\bmod 3)$ và $2016 \equiv 0(\bmod 7)$ nên ta có $x \neq 2016$. Vậy sau một số lần cắt, số miếng giấy Nam có được không thể bằng $2016 .$

 

 

 

 

 

 

 

 

 

 

 

 

 

 

 

 

 

 

 

Đề thi học kì 1 lớp 10 chuyên toán PTNK năm 2016

Thời gian làm bài: 120 phút

Câu 1.
a) Giải phương trình $x^{2}-x+2-(x+2) \sqrt{x-1}=0$.
b) Tìm $m$ để hệ phương trình $\left\{\begin{array}{l}x+y+x y=m \\ x^{2}+y^{2}=m\end{array}\right.$ có nghiệm.

Câu 2. Cho hàm số $y=f(x)=-x^{2}+2 x+3(1)$.
a) Khảo sát và vẽ đồ thị hàm số (1).
b) Từ đồ thị hàm số $(1)$, suy ra đồ thị hàm số $y=g(x)=-x^{2}+2|x|+3$. Tìm $k$ để phương trình $g(x)=m^{3}-3 m^{2}+m$ có đúng 3 nghiệm.

Câu 3.
a) Tìm giá trị lớn nhất và giá trị nhỏ nhất của hàm số
$$
y=\sqrt{x+1}+\sqrt{1-x}-\frac{4}{3} \sqrt{1-x^{2}}
$$
b) Cho các số $a, b, c>0$. Chứng minh rằng
$$
\left(\frac{a}{b}+\frac{b}{c}+\frac{c}{a}\right)^{2} \geq \frac{3}{2}\left(\frac{b+c}{a}+\frac{a+c}{b}+\frac{a+b}{c}\right)
$$
Bài 4. Cho tam giác $A B C$ cân tại $A, \angle B A C=120^{\circ}$ nội tiếp đường tròn tâm $O$ bán kính $R . A O$ cắt $(O)$ tại $D .$
a) Chứng minh rằng với mọi $M$ thì $\overrightarrow{M B} \cdot \overrightarrow{M C}=\overrightarrow{M A} \cdot \overrightarrow{M O}-\frac{R^{2}}{2}$.
b) Tìm quỹ tích điểm $M$ sao cho $\overrightarrow{M B} \cdot \overrightarrow{M C}-\overrightarrow{M A} \cdot \overrightarrow{M D}=\frac{R^{2}}{4}$.
c) Xác định điểm $N$ trên cạnh $B D$ thỏa $P_{D /(A B N)}=R^{2}$.
d) $P$ là điểm thay đổi trên cạnh $B C .$ Gọi $\left(O_{1}\right)$ là đường tròn qua $P$ tiếp xúc với $(O)$ tại $B ;\left(O_{2}\right)$ là đường tròn qua $P$ tiếp xúc với $(O)$ tại $C .\left(O_{1}\right)$ và $\left(O_{2}\right)$ cắt nhau tại $Q$ khác $P$. Chứng minh đường thẳng $P Q$ đi qua một điểm cố định $T$. Tính $P_{T /(O)}$.
Kí hiệu $P_{M /(O)}$ là phương tích của $M$ đối với đường tròn $(O)$.

Đề thi Học kì 1 Toán 10 PTNK năm 2016 (CS1)

Đề và lời giải: Thầy Nguyễn Tấn Phát

Bài 1. (1 điểm) Tìm m để phương trình $\dfrac{(x-1)(x-3m)}{\sqrt{x-2}+1}=0$ vô nghiệm
Bài 2. (1 điểm) Gọi $(P)$ là đồ thị của hàm số: $y= x^2 + bx + c \, \, (b,c \in \mathbb{R} )$. Biết các điểm $A(1;-4)$, $B(2;-3)$, thuộc $(P)$. \
Tìm tọa độ giao điểm của $(P)$ và $(P’)$, với $(P’)$ là đồ thị của hàm số $y= (2x-1)^2 -4$
Bài 3. (1 điểm) Cho hệ phương trình: $\left\{ \begin{array}{l}
x+\dfrac{1}{m} \sqrt{y} =4 \
\dfrac{1}{m} x + \sqrt{y} = \dfrac{2}{m} + 2
\end{array} \right.$, với m là tham số và $m \ne 0$. Định m để hệ phương trình có nghiệm duy nhất.
Bài 4. (2 điểm) Giải các phương trình sau:
a) $\sqrt{2x+1}+\sqrt{x-3}=4$
b) $x+ \dfrac{3x}{\sqrt{x^2-9}}=\dfrac{35}{4}$
Bài 5. (1 điểm) Chứng minh đẳng thức: $\tan^2 a – \tan^2 b = \dfrac{\sin(a+b).\sin(a-b)}{\cos^2a.\cos^2b}$
Bài 6. (1 điểm) Cho tam giác $ABC$ có các đỉnh $A(-1;3)$, $B(-3;-3)$, $C(2;2)$. Chứng minh tam giác $ABC$ là tam giác vuông và tìm trực tâm tam giác $ABC$.
Bài 7. (3 điểm) Cho hình bình hành $ABCD$ với $AB=6a$, $AD=3a$, $\angle ABC =60^0$. Gọi $M,N$ thỏa: $\overrightarrow{MA}+2 \overrightarrow{MB}=\overrightarrow{0}$, $3 \overrightarrow{ND}+2 \overrightarrow{NC}=\overrightarrow{0}$.
a) Tính $\overrightarrow{AM}. \overrightarrow{AD}$.
b) Tính độ dài cạnh $AN$ theo $a$.
c) Gọi $G$ là trọng tâm tam giác $AMN$. Tìm $x$ và $y$ thỏa: $\overrightarrow{BG}= x \overrightarrow{BA} + y \overrightarrow{BD}$.

Lời giải
Bài 1. (1 điểm)
Điều kiện: $x \ge 2$
$\dfrac{(x-1)(x-3m)}{\sqrt{x-2}+1}=0$ (1)
$\Leftrightarrow (x-1)(x-3m)=0$
$\Leftrightarrow x-3m=0$ (2) vì $x \ge 2$
Để phương trình (1) vô nghiệm thì (2) phải vô nghiệm $\Leftrightarrow 3m<2 \Leftrightarrow m < \dfrac{2}{3}$

Bài 2. (1 điểm)
$A(1;-4)$, $B(2;-3)$ thuộc $(P)$, ta có:
$\left\{ \begin{array}{l}
1+b+c=-4\\
4+2b+c=-3
\end{array} \right.$
$\Leftrightarrow \left\{ \begin{array}{l}
b=-2\\
c=-3
\end{array} \right.$
Do đó $(P): y=x^2 – 2x -3$
Phương trình hoành độ giao điểm của $(P)$ và $(P’)$: $ x^2-2x-3=(2x-1)^2 -4$
$\Rightarrow\left[ \begin{array}{l}
x=0 \Rightarrow y=-3 \\
x=\dfrac{2}{3} \Rightarrow y=-\dfrac{35}{9}
\end{array} \right.$
Vậy giao điểm của $(P)$ và $(P’)$ là $(0;-3)$ và $\left( \dfrac{2}{3};-\dfrac{35}{9} \right)$

Bài 3. (1 điểm)
Ta có:
$D=1-\dfrac{1}{m^2}$
$Dx= \dfrac{4m^2-2m-2}{m^2}$
$Dy= \dfrac{2m-2}{m}$
Để hệ phương trình có nghiệm duy nhất thì:
$\left\{ \begin{array}{l}
D \ne 0 \\
y=\dfrac{Dy}{D} \ge 0
\end{array} \right.$
$\Rightarrow \left\{ \begin{array}{l}
m \ne \pm 1\\
\left[ \begin{array}{l}
m \ge 0 \\
m \le -1
\end{array} \right.
\end{array} \right. $
$\Rightarrow \left[ \begin{array}{l}
m < -1 \\ \left\{ \begin{array}{l} m > 0 \\
m \ne 1
\end{array} \right.
\end{array} \right.$

Bài 4. (2 điểm)
a) Điều kiện: $x \ge 3$.
Ta có $\sqrt{2x+1}+\sqrt{x-3} = 4 \Leftrightarrow 2\sqrt{2x^2-5x-3}=18 – 3x$
$\Leftrightarrow 4(2x^2-5x-3) = 9x^2-108x + 324$ ($x\leq 6$)
$\Leftrightarrow x^2-88x+332 = 0 \Leftrightarrow x = 4 (n), x = 84(l)$.
Vậy $S=\{4\}$.
b) Điều kiện: $x^2 \ge 9 \Rightarrow \left[ \begin{array}{l}
x \ge 3 \\
x \le -3
\end{array} \right.$
$x+ \dfrac{3x}{\sqrt{x^2-9}}=\dfrac{35}{4}$
$\Rightarrow x^2 + \dfrac{9x^2}{x^2-9} + \dfrac{6x^2}{\sqrt{x^2-9}}=\left( \dfrac{35}{4} \right)^2$
$\Rightarrow \dfrac{x^4}{x^2-9}+\dfrac{2.x^2.3}{\sqrt{x^2-9}} +9 = \left( \dfrac{35}{4} \right)^2 + 9$
$\Rightarrow \left( \dfrac{x^2}{\sqrt{x^2-9}} +3 \right)^2 = \dfrac{1369}{16}$
$\Rightarrow \left[ \begin{array}{l}
\dfrac{x^2}{\sqrt{x^2-9}} +3 = \dfrac{37}{4} \\
\dfrac{x^2}{\sqrt{x^2-9}} +3 = -\dfrac{37}{4} \quad \text{(loại)}
\end{array} \right.$
$\Rightarrow \dfrac{x^2}{\sqrt{x^2-9}} = \dfrac{25}{4}$
$\Rightarrow 16x^4=625.x^2-9.625$
$\Rightarrow \left[ \begin{array}{l}
x= \pm 5\\
x= \pm \dfrac{15}{4}
\end{array} \right.$
Thử lại nghiệm ta chọn $x=5$ hoặc $x=\dfrac{15}{4}$
Vậy $x=5$ hoặc $x=\dfrac{15}{4}$

Bài 5. (1 điểm)
$\tan^2 a – \tan^2 b \\
= \dfrac{\sin^2a}{\cos^2a}-\dfrac{\sin^2b}{\cos^2b} \\
= \dfrac{\sin^2a.\cos^2b – \sin^2b.\cos^2a}{\cos^2a.\cos^2b} \\
= \dfrac{(\sin a.\cos b+\sin b.\cos a)(\sin a.\cos b-\sin b.\cos a )}{\cos^2a.\cos^2b} \\
= \dfrac{\sin(a+b).\sin(a-b)}{\cos^2a.\cos^2b}$
Bài 6. (1 điểm)
$AB^2 =(-3+1)^2+(-3-3)^2=40$
$BC^2 =(2+3)^2+(2+3)^2=50$
$AC^2 =(2+1)^2+(2-3)^2=10$
$\Rightarrow BC^2=AB^2+AC^2$
Vậy tam giác $ABC$ vuông tại $A$ và có $A$ là trực tâm.

Bài 7. (3 điểm)
a) $MA = \dfrac{2}{3}AB = 4a$
$\angle BAD = 180^0-\angle ABC = 180^0-60^0 = 120^0 $
$\overrightarrow{AM}.\overrightarrow{AD}=|\overrightarrow{AM}|.|\overrightarrow{AD}|.\cos(\overrightarrow{AM},\overrightarrow{AD}) = 4a.3a.\cos 120^0 = -6a^2$
b) $DN = \dfrac{2}{3} CN=\dfrac{2}{5} CD
= \dfrac{2}{5}.6a=\dfrac{12}{5}a$
Áp dụng định lý cosin cho tam giác $ADN$, ta có:
\begin{align*}
AN^2&= AD^2+DN^2-2.AD.DN.\cos \angle ADN \\
&= (3a)^2 + \left( \dfrac{12}{5}a \right)^2 – 2.3a.\dfrac{12}{5}a.\cos 60^0 \\
&= \dfrac{189}{25} a^2
\end{align*}
Do đó $AN= \dfrac{3\sqrt{21}}{5}a$
c) \begin{align*}
\overrightarrow{BG} &= \overrightarrow{BE} + \overrightarrow{EG} \\
&= \dfrac{2}{3}\overrightarrow{BA} + \dfrac{1}{3} \overrightarrow{EN} \\
&= \dfrac{2}{3}\overrightarrow{BA} + \dfrac{1}{3} \left( \overrightarrow{EA}+\overrightarrow{AN} \right) \\
&= \dfrac{2}{3}\overrightarrow{BA} + \dfrac{1}{3}.\dfrac{1}{3}\overrightarrow{BA}+ \dfrac{1}{3} \left( \overrightarrow{AD} + \overrightarrow{DN} \right) \\
&= \dfrac{2}{3}\overrightarrow{BA} + \dfrac{1}{9} \overrightarrow{BA} + \dfrac{1}{3} \left( \overrightarrow{BD} – \overrightarrow{BA} \right) + \dfrac{1}{3}.\left( -\dfrac{2}{5} \right) \overrightarrow{BA} \\
&= \dfrac{14}{45}\overrightarrow{BA} + \dfrac{1}{3} \overrightarrow{BD}
\end{align*}
Vậy $x=\dfrac{14}{45}$ và $y=\dfrac{1}{3}$

Đề thi và đáp án kì thi chọn đội tuyển thi Quốc gia trường Phổ thông Năng khiếu năm học 2016 -2017

Đề thi

Ngày thi thứ nhất

Bài 1. Tìm tất cả $a$ để dãy số $(u_n)$ hội tụ, biết $u_1=a$ và $\forall n\in \mathbb{N}^*$ thì:
$$u_{n+1}=\left\{\begin{array}{l}
2u_n-1\ \text{nếu $u_n>0$,}\\
-1\ \text{nếu $-1\le u_n\le 0$,}\\
u_n^2+4u_n+2\ \text{nếu $u_n<-1$.}
\end{array} \right.$$

Bài 2. Tìm số nguyên dương $k$ nhỏ nhất để bất đẳng thức $$x^ky^kz^k(x^3+y^3+z^3)\le 3$$
luôn đúng với mọi số thực dương $x,y,z$ thoả mãn điều kiện $x+y+z=3$.

Bài 3. Cho hàm số $f:\mathbb N^* \rightarrow \mathbb N^*$ thoả mãn hai điều kiện sau:

i)  $f$ là hàm tăng thật sự trên $\mathbb N^*$.

ii) $f(2n)=2f(n)\ \forall n\in \mathbb N^*$.

a) Giả sử $f(1)=3$ và $p>3$ là số nguyên tố. Chứng minh rằng tồn tại số nguyên dương $n$ sao cho $f(n)$ chia hết cho $p$.
b) Cho $q$ là số nguyên tố lẻ. Hãy xây dựng một hàm $f$ thoả mãn các điều kiện của bài toán mà $f(n)$ không chia hết cho $q$ với mọi $n$ nguyên dương.

Bài 4. Cho tam giác $ABC$ có góc $\angle BAC$ tù và $AH\perp BC$ ($H$ nằm trên $BC$). Điểm $M$ thay đổi trên cạnh $AB$. Dựng điểm $N$ sao cho $\Delta BMN\sim \Delta HCA$, với $H$ và $N$ nằm khác phía đối với đường thẳng $AB$.

a) Gọi $CM$ cắt đường tròn ngoại tiếp tam giác $BMN$ tại $K$. Chứng minh rằng $NK$ luôn đi qua một điểm cố định.
b) Gọi $NH$ cắt $AC$ tại $P$. Dựng điểm $Q$ sao cho $\triangle HPQ\sim \Delta HNM$, với $Q$ và $M$ nằm khác phía đối với đường thẳng $NP$. Chứng minh rằng $Q$ luôn thuộc một đường thẳng cố định.

Ngày thi thứ hai

Bài 5. Với mỗi số nguyên dương $n$, tồn tại duy nhất số tự nhiên $a$ thoả mãn điều kiện $a^2\le n<(a+1)^2$. Đặt $\Delta_n=n-a^2$.

a) Tìm giá trị nhỏ nhất của $\Delta_n$ khi $n$ thay đổi và luôn thoả mãn $n=15m^2$ với $m$ là số nguyên dương.
b) Cho $p,q$ là các số nguyên dương và $d=5(4p+3)q^2$. Chứng minh rằng $\Delta_d\ge 5$.

Bài 6.  Với các số nguyên $a,b,c,d$ thoả mãn $1\le a<b<c<d$, ký hiệu:
$$T(a,b,c,d)={{x,y,z,t}\subset \mathbb{N}^*\mid 1\le x<y<z<t,\ x\le a,y\le b,z\le c,t\le d}$$

a) Tình số phần tử của $T(1,4,6,7)$.
b) Cho $a=1$ và $b\ge 4$. Gọi $d_1$ là số phần tử của $T(a,b,c,d)$ chứa $1$ và không chứa $2$; $d_2$ là số phần tử chứa $1,2$ và không chứa $3$; $d_3$ là số phần tử chứa $1,2,3$ và không chứa $4$. Chứng minh rằng $d_1\ge 2d_2-d_3$. Đẳng thức xảy ra khi nào ?

Bài 7. Trong một hệ thống máy tính, một máy tính bất kỳ có kết nối trực tiếp với ít nhất $30\%$ máy tính khác của hệ thống. Hệ thống này có một chương trình cảnh báo và ngăn chặn khá tốt, do đó khi một máy tính bị virus, nó chỉ có đủ thời gian lây cho các máy tính được kết nối trực tiếp với nó. Chứng minh rằng dù vậy, kẻ tấn công vẫn có thể chọn hai máy tính của hệ thống mà nếu thả virus vào hai máy đó, ít nhất $50\%$ máy tính của hệ thống sẽ bị nhiễm virus.

Bài 8. Cho tam giác $ABC$ nhọn. Đường tròn $(I)$ có tâm $I$ thuộc cạnh $BC$ và tiếp xúc với các cạnh $AB,AC$ lần lượt tại $E,F$. Lấy $M,N$ bên trong tứ giác $BCEF$ sao cho $EFNM$ nội tiếp $(I)$ và các đường thẳng $MN,EF,BC$ đồng quy. Gọi $MF$ cắt $NE$ tại $P$, $AP$ cắt $BC$ tại $D$.

a) Chứng minh rằng $A,D,E,F$ cùng thuộc một đường tròn.
b) Lấy trên các đường thẳng $BN,CM$ các điểm $H,K$ sao cho $\angle ACH=\angle ABK=90^\circ$. Gọi $T$ là trung điểm $HK$. Chứng minh rằng $TB=TC$.

Hết

Lời giải

Lời giải

Bài 1. 

  • Nếu $a>1$, bằng quy nạp đơn giản, ta có $u_n>1\ \forall n\in \mathbb N^*$ và
    $$u_n = 2^{n-1}(a-1)+1, \ \forall n\in \mathbb N^*.$$
    Do $a>1$, cho $n\rightarrow +\infty$ thì $u_n\rightarrow +\infty$. Từ đó $(u_n)$ không hội tụ.
  •  Nếu $a=1$ thì $u_n=1\ \forall n\in \mathbb N^*$ hay $(u_n)$ hội tụ về $1$.
  • Nếu $0<a<1$, ta sẽ chứng minh rằng $(u_n)$ có ít nhất một số hạng không dương. Thật vậy, giả sử $u_n>0\ \forall n\in \mathbb N^*$ thì theo trường hợp đầu tiên, ta có:
    $$u_n = 2^{n-1}(a-1)+1\ \forall n\in \mathbb N^*$$
    Do $a>1$, cho $n\rightarrow +\infty$ thì $u_n\rightarrow -\infty$, trái với việc $u_n>0\ \forall n, \in \mathbb N^*$. Từ đó điều giả sử là sai hay phải tồn tại $k\in \mathbb N^*$ sao cho $u_k>0$ và $u_{k+1}\le 0$. Với cách chọn chỉ số $k$ như vậy, ta có:
    $$-1\le 2u_k-1=u_{k+1}\le 0$$
    Khi đó $u_{k+2}=0$. Bằng quy nạp thì $u_n=-1\ \forall n\in \mathbb N^*, n\ge k+2$. Điều này dẫn đến $(u_n)$ hội tụ về $-1$.
  • Nếu $-1\le a\le 0$, từ giả thiết thì $u_2=-1$. Bằng quy nạp thì $u_n=-1\ \forall n\in \mathbb N^*, n\ge 2$ hay $(u_n)$ hội tụ về $-1$.
  • Nếu $-2<a<-1$, ta có:
    $$u_2-u_1=a^2+3a+2=(a+2)(a+1)<0$$
    Khi đó thì $u_2<u_1<-1$. Lại có $u_2=(a+2)^2-2\ge -2$ nên $-2<u_2<-1$. Bằng quy nạp, ta có $(u_n)$ là dãy giảm và $-2<u_n<-1$ nên $(u_n)$ hội tụ.
  • Nếu $-2-\sqrt{3}\le a\le -2$ thì $u_2=a^2-4a+2$ và dễ có được:
    $$-1\le a^2-4a+2\le 1$$
    Theo các trường hợp đã xét, dãy số $(u_n)$ hội tụ.
  • Nếu $a<-2-\sqrt{3}$, bằng vài tính toán, ta có $u^2=a^2-4a+2>1$.\\
    Theo trường hợp đầu tiên, dãy số $(u_n)$ không hội tụ.Vậy dãy số $(u_n)$ hội tụ khi và chỉ khi $-2-\sqrt{3}\le a\le 1$.]

Bài 2. Ta sẽ chứng minh rằng $k=3$ là số nguyên dương nhỏ nhất thoả mãn bài toán. Trước hết, chọn $x=y=\dfrac{3}{4},z=\dfrac{3}{2}$ thì ta phải có:
$$\left(\frac{3}{4}\right)^{2k}\cdot \left(\frac{3}{2}\right)^{k}\left(2\cdot\left(\frac{3}{4}\right)^{3}+\left(\frac{3}{2}\right)^3\right)\le 3$$
Dễ thấy đánh giá trên chỉ đúng nếu $k\ge 3$. Ta đưa về chứng minh rằng:
$$x^3y^3z^3(x^3+y^3+z^3)\le 3.$$
Không mất tính tổng quát, giả sử $x\ge y\ge z$ thì $z \le 1$. Ta có:
$$\begin{aligned} & x^3+y^3=(x+y)^3-3xy(x+y)=(3-z)^3-3xy(x+y) \text{ hay} \\
&(3-z)^3 + z^3 \le \frac{3}{x^3y^3z^3}+3xy(x+y). \end{aligned} $$
Khai triển và thu gọn, bất đẳng thức trở thành:
$$3z^2-9z+9 \le \frac{1}{x^3y^3z^3}+x^2y+xy^2.$$
Theo bất đẳng thức AM-GM, ta có vế phải của bất đẳng thức trên sẽ không nhỏ hơn $\frac{3}{z}$. Từ đây ta chỉ cần chứng minh rằng $$3z^2-9z+9 \le \dfrac{3}{z} \text{ hay } 3(z-1)^3 \le 0, \text{ đúng.}$$
Vậy $k=3$ là hằng số nguyên dương nhỏ nhất thoả mãn bài toán.

Bài 3.

(a) Đặt $A = \\{f(n+1)-f(n)|n \in\mathbb{N^{*}}\\}$.

Vì $f$ là hàm số tăng thực sự trên $\mathbb{N^{*}}$ nên $A\subset \mathbb{N^{*}}$.

Khi đó phải tồn tại $k=\min A$ và tồn tại $n\in \mathbb N^*$ để $k=f(n+1)-f(n)$.

Khi đó:
$$f(2n+2) – f(2n) = 2f(n+1) – 2f(n) = 2k.$$

Lại có $f(2n+2) – f(2n+1),f(2n+1) – f(2n)\ge k$ nên

$$f(2n+2)-f(2n+1)+f(2n+1)-f(2n)\ge 2k.$$
Từ đây ta phải có $f(2n+2) – f(2n+1)=f(2n+1) – f(2n)= k$. Bằng quy nạp theo $m$, ta chứng minh được

$$f(2^mn+t) = 2^mf(n)+tk\ \forall t,m\in \mathbb N, t\le m.$$

Lại có $f(1)=3,f(2)=6$ nên $k \le 3<p$ hay $(k, p)=1$. \medskip

Xét $p$ số nguyên dương sau:
$$f(2^pn), f(2^pn+1), f(2^pn+2),\ldots, f(2^pn+p-1)$$
lập thành một cấp số cộng có công sai $k$ nên là một hệ thặng dư đầy đủ modulo $p$. Từ đó phải tồn tại một số hạng chia hết cho $p$. \medskip

(b) Ta xây dựng một hàm số $f$ với các điều kiện như sau:

$f(1) = 2^a > q (a\in\mathbb{N^{*}},$

$f(2n)=2f(n)\ \forall n\in \mathbb{N^{*}},$

$f(2n+1)=f(2n)+q\ \forall n\in \mathbb{N^{*}}.$

Ta chứng minh rằng hàm số $f$ vừa xây dựng thỏa mãn bài toán. \medskip

Trước hết ta chứng minh rằng $f$ là hàm tăng thực sự, cụ thể là:
$$f(n+1) – f(n) \geq q\ \forall n\in \mathbb{N^{*}}.$$
Với $n = 1$, ta có $f(2)-f(1) = 2.2^a – 2^a = 2^a > q$. Giả sử khẳng định cần chứng minh đúng đến $n=k$. Xét các khả năng sau:

Nếu $k$ là số chẵn, ta có $f(k+1)=f(k)+q$ thỏa mãn yêu cầu.
Nếu $k$ là số lẻ, ta có:
$$f(k+1)= 2f\left(\dfrac{k+1}{2}\right) \geq 2\left(f\left(\dfrac{k-1}{2}\right)+q\right)= f(k-1)+2q.$$
Lại có $f(k)=f(k-1)+q$ nên $f(k+1)\ge f(k)+q$.

Theo nguyên lý quy nạp, ta có $f(n+1) – f(n) \geq q\ \forall n\in \mathbb{N^{*}}$. \medskip

Bây giờ ta chứng minh rằng không tồn tại $n$ để $q \mid f(n)$. Trước hết thì $f(1) = 2^a$ không chia hết cho $q$. Giả sử điều này đúng đến $n=k$. Xét các khả năng sau:

Nếu $k$ chẵn thì $f(k+1)=f(k)+q$ không chia hết cho $q$.
Nếu $k$ lẻ thì $f(k+1)= 2f\left(\dfrac{k+1}{2}\right)$ không chia hết cho $q$.

Theo nguyên lý quy nạp, $f(n)$ không chia hết cho $q$ với mọi $n\in \mathbb{N^{*}}$.
Các điều kiện đã được kiểm tra đầy đủ.

BÀI 4.

(a) Ta sẽ chứng minh rằng $AD\perp BC$. Gọi $X$ là điểm đồng quy của $EF,MN,BC$. Do $AE,AF$ tiếp xúc với $(I)$ nên $EF$ là đường đối cực của $A$ đối với $(I)$. Ta có $X\in EF$ nên theo định lý La Hire, điểm $A$ sẽ nằm trên đường đối cực của $X$ đối với đường tròn $(I)$. \medskip

Lại có $P$ là giao điểm của $EN,FM$ nên $P$ nằm trên đường đối cực của $X$ đối với $(I)$. Vì thế nên $AP$ là đường đối cực của $X$ đối với $(I)$ hay $AP\perp BC$. Do đó $$\angle ADI=\angle AEI=\angle AFI=90^\circ.$$
Vậy $A,D,E,F$ cùng thuộc một đường tròn.

(b) Gọi $S$ là giao điểm của $BN,CM$. Xét hai tam giác $PEF,SBC$ có $PE$ cắt $SB$ tại $N$, $PF$ cắt $SC$ tại $M$, $EF$ cắt $BC$ tại $X$ và $X,M,N$ thẳng hàng. Theo định lý Desargues thì $PS,EB,FC$ đồng quy. Mặt khác $EB$ cắt $FC$ tại $A$ nên $A,P,S$ thẳng hàng, dẫn đến $S\in AD$. \medskip

Tiếp theo ta sẽ chứng minh rằng $\angle BAK=\angle CAH$. Áp dụng định lý Ceva dạng lượng giác cho tam giác $ABC$ với:

Các đường thẳng $AD,BH,CK$ đồng quy:
$$\frac{\sin\angle DAB}{\sin \angle DAC}\cdot \frac{\sin\angle HBC}{\sin \angle HBA}\cdot \frac{\sin\angle KCA}{\sin \angle KCB}=1$$
Các đường thẳng $AH,BH,CH$ đồng quy:
$$\frac{\sin\angle HAB}{\sin \angle HAC}\cdot \frac{\sin\angle HBC}{\sin \angle HBA}\cdot \frac{\sin\angle HCA}{\sin \angle HCB}=1$$
Các đường thẳng $AK,BK,CK$ đồng quy:
$$\frac{\sin\angle KAB}{\sin \angle KAC}\cdot \frac{\sin\angle KBC}{\sin \angle KBA}\cdot \frac{\sin\angle KCA}{\sin \angle KCB}=1$$

Chú ý rằng do các góc vuông và góc bù nhau nên ta có
$$\frac{\sin\angle HAC}{\sin \angle HAB}=\frac{\sin\angle KAB}{\sin \angle KAC}$$
Từ đó sử dụng công thức cộng cho mẫu thức và biến đổi thì:
$$\tan\angle HAC=\tan\angle KAB$$
Dẫn đến $\angle HAC=\angle KAB$. Cuối cùng, ta sẽ chứng minh $TB=TC$.
Gọi $U,V$ lần lượt là trung điểm của các đoạn $AK,AH$. Ta có:
$$UB=\dfrac{AK}{2}=VT,UT=\dfrac{AH}{2}=VC.$$
Đồng thời, ta cũng có:
$$\angle BUT=\angle BUA-\angle AUT=\angle AVC-\angle AVT=\angle TVC$$
Do đó $\Delta BUT=\Delta TVC$ (c.g.c), vậy nên $TB=TC$.

Bài 5. 

(a) Ta cần tìm $\Delta_n$ nhỏ nhất để phương trình $15m^2 – a^2 = \Delta_n$ có nghiệm nguyên dương. Nhận thấy $15 – 3^2 = 6$ nên $\min \Delta_n\le 6$. Ta chứng minh rằng phương trình trên không có nghiệm nguyên dương với $\Delta_n < 6$. \medskip

Ta có $3\mid a^2 + \Delta_n$. Suy ra $3\mid \Delta_n $ hoặc $3\mid \Delta_n+1$. Mặt khác $5\mid a^2 + \Delta_n$ nên $\Delta_n$ chia $5$ chỉ có thể dư $0,1$ hoặc $4$. \medskip

Từ đó nếu tồn tại $n$ để $\Delta_n< 6$ thỏa mãn bài toán thì $\Delta_n = 5$. Giả sử rằng tồn tại $n$ như thế, ta có $15m^2-a^2=5$ hay $5\mid a$. Đặt $a=5s$ $(s\in \mathbb N^*)$, ta có:
$$3m^2 – 5s^2 = 1.$$
Từ đó thì $$3(m^2+s^2)\equiv 1 \pmod{8} \text{ hay } m^2+s^2\equiv 3 \pmod{8}.$$
Điều này vô lý do $m^2$ chia $8$ dư $0,1,4$. Vậy $\Delta_n$ nhỏ nhất là $6.$ \medskip

(b) Ta có $$5(4p+3)q^2-a^2=\Delta_d.$$ Do $a^2$ chia $5$ dư $0,1,4$ nên $\Delta_d$ chia $5$ dư $0,1,4$. Giả sử rằng có bộ số để $\Delta_d<5$. Xét các khả năng sau:

Nếu $\Delta_d=0$ thì $5(4p+3)q^2=a^2$. Xét bộ số $(q,a)$ với $q+a$ nhỏ nhất. Từ phương trình trên, ta có $a^2+q^2\equiv 0$ (mod $4$) hay $a\equiv q\equiv 0$ (mod $2$).\medskip

Đặt $a=2a_1$ và $q=2q_1$ với $a_1,q_1\in \mathbb N^*$ thì bộ số $(q_1,a_1)$ cũng thoả mãn điều kiện $5(4p+3)q_1^2=a_1^2$. Hơn nữa $q_1+a_1<q+a$, mâu thuẫn.
Nếu $\Delta_d = 1$, ta có $a^2 + 1 = 5(4p+3)q^2$. Do $5(4p+3)\equiv 3$ (mod $4$) nên số này tồn tại một ước nguyên tố $r\equiv 3$ (mod $4$).\\
Do đó $a^2+1\equiv 0$ (mod $r$) hay $r\mid 1$, vô lý.
Nếu $\Delta_d = 4$, chứng minh tương tự, ta cũng có điều mâu thuẫn.

Vậy ta phải có $\Delta_d \ge 5$.

Bài 6.

(a) Với $T(1,4,6,7)$, ta có $x \leq 1$ nên $x =1$. Khi đó ta có $2\le y \le 4$ hay $y\in \{2,3,4\}$. Xét các khả năng sau:

Nếu $y = 2$ thì $3\leq z \leq 6$. Với mỗi giá trị của $z$, ta có thể thu được $7-z$ giá trị của $t$ nên ta có 10 bộ số.
Nếu $y=3$, tương tự ta có $6$ bộ số.
Nếu $y= 4$, tương tự ta có $3$ bộ số.

Vậy có tất cả $19$ bộ số trong $T(1,4,6,7)$. \medskip

(b) Đặt các tập hợp sau:
$$\begin{cases}
T_1 = \{(1, y, z, t)\mid 3\le y \le b, y<z\le c, z<t \leq d \}\\\\
T_2 = \{(1, 2, z, t)\mid 4\le z \le c, z<t\le d \}\\\\
T_3 = \{(1, 2, 3, t)\mid 5\le t \le d \}
\end{cases}.$$
Ta có $d_3 = |T_3| = d – 4$ và
$$d_2=\sum_{z=4}^{c}(d-z)=(c-3)d+\frac{(c+4)(c-3)}{2}.$$
Tiếp theo ta tính $d_1=|T_1|$. Vì $b \ge 4$ nên $y \ge 3$. Xét các khả năng sau

Nếu $y=3$ thì $T(1,3,z,t)=d_2$.
Nếu $y=4$ thì $T(1,4,z,t)=\sum_{z=5}^{c} (d-z)=(c-4)d-\dfrac{(c+5)(c-4)}{2}$. \medskip

Từ đó $d_1\ge d_2+(c-4)d-\dfrac{(c+5)(c-4)}{2}$. Do đó, kết hợp với việc tính được giá trị của $d_2$, khi cộng theo vế thì $d_1+d_3 – 2d_2 \ge 0.$

Vậy $d_1\ge 2d_2-d_3$. Đẳng thức xảy ra khi và chỉ khi $b=4$.

Ngoài lời giải khá “đại số” phía trên, có một lời giải khác cho ý sau của bài toán sử dụng song ánh:

Điểm mấu chốt là phân rã $T_1,T_2,T_3$ thành các nhóm thích hợp và thiết lập được đơn ánh giữa chúng. Với các tập $T_1,T_2,T_3$ định nghĩa như trên, ta viết $T_1$ thành $A\cup B\cup C$ có giao đôi một khác rỗng, trong đó
$$\begin{cases}
A = \{(1, 3, 4, t)\mid 5\le t \le d \}\\\\
B = \{(1, 3, z, t)\mid 5\le z \le c, z<t\le d \}\\\\
C = \{(1, y, z, t)\mid 4\le y\le b, y<z\le c, z<t \le d \}
\end{cases}.$$
Dễ kiểm chứng rằng có song ánh từ $A$ vào $T_3$ nên $|A|=|T_3|=d_3$.
Xét $D=\{(1, 4, z, t)\mid 5\le z \le c, z<t\le d \}$. Dễ kiểm chứng rằng $D\subset C$ và có song ánh từ $D$ vào $B$ nên $|D|=|B|$.
Ta có $A\cup B=\{(1, 3, z, t)\mid 4\le z \le c, z<t\le d \}$. Dễ kiểm chứng rằng có song ánh từ $A\cup B$ vào $T_2$ nên $|A\cup B|=|T_2|=d_2$. Chú ý rằng $A\cap B=\varnothing$ nên $|A|+|B|=d_2$ hay $|B|=d_2-d_3$. Từ đó ta có:
$$d_1=|A|+|B|+|C|\ge |A|+|B|+|D|=d_3+2|B|$$
Vậy $d_1\ge d_3+2(d_2-d_3)=2d_2-d_3$. Đẳng thức xảy ra khi và chỉ khi $b=4$.

Bài 7.

Trước hết ta chứng minh bổ đề sau: Xét một tập con $S$ bất kỳ của tập các máy tính $X$, khi đó tồn tại $1$ máy tính của hệ thống kết nối trực tiếp với ít nhất $30\%$ máy tính của $S$. \medskip

Thật vậy, xét các cặp $(s, x)$ với $s\in S,x\in X$ và $(s,x)$ kết nối trực tiếp với nhau. Khi đó, nếu tính theo $s$ thì số cặp như vậy sẽ không ít hơn $\dfrac{3}{10}|S||X|$. Do đó nếu tính theo $x$ thì sẽ phải tồn tại máy tính $x$ kết nối trực tiếp với ít nhất $\dfrac{3}{10}|S|$. \medskip

Quay trở lại bài toán, \medskip

Giả sử hệ thống có $n$ máy tính. Xét máy tính $A$ bất kỳ. Gọi $S$ là tập hợp các máy tính không kết nối trực tiếp với $A$. Nếu $S=\varnothing$ thì kết quả bài toán là hiển nhiên. Nếu $S\ne \varnothing$ thì theo bổ đề, tồn tại máy tính $B$ kết nối trực tiếp với ít nhất $30\%$ máy tính trong $S$. Ta chứng minh hai máy tính $A$ và $B$ thỏa mãn yêu cầu bài toán. \medskip

Thật vậy, giả sử $A$ kết nối trực tiếp với $k$ máy tính khác. Khi đó, theo cách chọn, $A$ và $B$ sẽ kết nối trực tiếp với ít nhất
$$k + 0,3(n-k) = 0,7k + 0,3n \ge 0,7\cdot 0,3n + 0,3n = 0,51n.$$
Từ đây ta có được kết luận của bài toán.

Bài 8.

(a) Ta sẽ chứng minh rằng $AD\perp BC$. Gọi $X$ là điểm đồng quy của $EF,MN,BC$. Do $AE,AF$ tiếp xúc với $(I)$ nên $EF$ là đường đối cực của $A$ đối với $(I)$. Ta có $X\in EF$ nên theo định lý La Hire, điểm $A$ sẽ nằm trên đường đối cực của $X$ đối với đường tròn $(I)$. \medskip

Lại có $P$ là giao điểm của $EN,FM$ nên $P$ nằm trên đường đối cực của $X$ đối với $(I)$. Vì thế nên $AP$ là đường đối cực của $X$ đối với $(I)$ hay $AP\perp BC$. Do đó $$\angle ADI=\angle AEI=\angle AFI=90^\circ.$$
Vậy $A,D,E,F$ cùng thuộc một đường tròn.

(b) Gọi $S$ là giao điểm của $BN,CM$. Xét hai tam giác $PEF,SBC$ có $PE$ cắt $SB$ tại $N$, $PF$ cắt $SC$ tại $M$, $EF$ cắt $BC$ tại $X$ và $X,M,N$ thẳng hàng. Theo định lý Desargues thì $PS,EB,FC$ đồng quy. Mặt khác $EB$ cắt $FC$ tại $A$ nên $A,P,S$ thẳng hàng, dẫn đến $S\in AD$. \medskip

Tiếp theo ta sẽ chứng minh rằng $\angle BAK=\angle CAH$. Áp dụng định lý Ceva dạng lượng giác cho tam giác $ABC$ với:

Các đường thẳng $AD,BH,CK$ đồng quy:
$$\frac{\sin\angle DAB}{\sin \angle DAC}\cdot \frac{\sin\angle HBC}{\sin \angle HBA}\cdot \frac{\sin\angle KCA}{\sin \angle KCB}=1$$
Các đường thẳng $AH,BH,CH$ đồng quy:
$$\frac{\sin\angle HAB}{\sin \angle HAC}\cdot \frac{\sin\angle HBC}{\sin \angle HBA}\cdot \frac{\sin\angle HCA}{\sin \angle HCB}=1$$
Các đường thẳng $AK,BK,CK$ đồng quy:
$$\frac{\sin\angle KAB}{\sin \angle KAC}\cdot \frac{\sin\angle KBC}{\sin \angle KBA}\cdot \frac{\sin\angle KCA}{\sin \angle KCB}=1$$

Chú ý rằng do các góc vuông và góc bù nhau nên ta có
$$\frac{\sin\angle HAC}{\sin \angle HAB}=\frac{\sin\angle KAB}{\sin \angle KAC}$$
Từ đó sử dụng công thức cộng cho mẫu thức và biến đổi thì:
$$\tan\angle HAC=\tan\angle KAB$$
Dẫn đến $\angle HAC=\angle KAB$. Cuối cùng, ta sẽ chứng minh $TB=TC$.

Gọi $U,V$ lần lượt là trung điểm của các đoạn $AK,AH$. Ta có:
$$UB=\dfrac{AK}{2}=VT,UT=\dfrac{AH}{2}=VC.$$
Đồng thời, ta cũng có:
$$\angle BUT=\angle BUA-\angle AUT=\angle AVC-\angle AVT=\angle TVC$$
Do đó $\Delta BUT=\Delta TVC$ (c.g.c), vậy nên $TB=TC$.

Đáp án thi chọn Đội Tuyển thi Quốc Gia của trường PTNK năm học 2015 – 2016

Ngày thứ 1

Bài 1. Cho tập hợp
$$
A=\{n \in \mathbb{N} \mid 1 \leq n \leq 2015,(n, 2016)=1\}
$$
Hỏi có bao nhiêu số nguyên $a \in A$ sao cho tồn tại số nguyên b mà $a+2016 b$ là số chính phương?

Bài 2. Cho $a, b, c, d$ là các số thực thỏa mãn điều kiện
$$
a^{2} \leq 1, a^{2}+b^{2} \leq 5, a^{2}+b^{2}+c^{2} \leq 14, a^{2}+b^{2}+c^{2}+d^{2} \leq 30
$$
1. Chúng minh rằng $a+b+c+d \leq 10$.
2. Chứng minh rằng $a d+b c \leq 10$.

Bài 3. Tìm tất cả các hàm số $f: \mathbb{R} \rightarrow \mathbb{R}$ thỏa mãn điều kiện
$$
f(x-2 f(y))=5 f(x)-4 x-2 f(y)
$$
với mọi $x, y \in \mathbb{R}$.

Bài 4. Cho đường tròn $k$ và các điểm $B, C$ thuộc đường tròn, không phải là đường kính; I là trung điểm $B C$. Điểm $A$ di động trên cung lớn $B C$ của $k$. Gọi $i_{1}$ là đường tròn qua $I$ và tiếp xúc với $A B$ tại $B ; i_{2}$ là đường tròn qua $I$ và tiếp xúc với $A C$ tại $C$. Các đường tròn $i_{1}, i_{2}$ cắt nhau tại $D$ (khác $I$ ).
1. Chứng minh rằng đường tròn ngoại tiếp tam giác AID luôn đi qua một điểm cố định.
2. Gọi $K$ là trung điểm $A D$, $E$ là tâm đường tròn qua $K$ và tiếp xúc với $A B$ tại $A, F$ là tâm đường tròn qua $K$ và tiếp xúc với AC tại $A$. Chứng minh rằng góc EAF có số đo không đổi.

Ngày thứ 2

Bài 5. Dãy số $\left(x_{n}\right)$ được xác định bởi công thức $x_{n}=\frac{1}{n \cos \frac{1}{n}}$ với mọi $n \geq 1$. Tính giới hạn sau
$$\lim \frac{x_{1}+x_{3}+x_{5}+\cdots+x_{2 n-1}}{x_{2}+x_{4}+x_{6}+s+x_{2 n}}$$

Bài 6. Tim các giá trị của $b$ sao cho tồn tại a để hệ phương trình sau có nghiệm $(x, y)$
$$
\left\{\begin{array}{l}
(x-1)^{2}+(y+1)^{2}=b \\y=x^{2}+(2 a+1) x+a^{2}
\end{array}\right.
$$

Bài 7. Cho n là số nguyên dương, $n \geq 2$ và $X={1,2,3, \ldots, n}$. Gọi $A_{1}, A_{2}, \ldots, A_{m}$ và $B_{1}, B_{2}, \ldots, B_{m}$ là hai dãy các tập con khác rỗng của $X$ thỏa mãn điều kiện: Với mỗi $i, j \in{1,2,3, \ldots, n}, A_{i} \cap B_{j}=\varnothing$ nếu và chỉ nếu $i=j$.
1. Chúng minh rằng với mỗi hoán vị $\left(x_{1}, x_{2}, \ldots, x_{n}\right)$ của $X$, có không quá một cặp tập hợp $\left(A_{i}, B_{i}\right)$ với $i=1,2,3, \ldots, n$ sao cho nếu $x_{k} \in A_{i}$ và $x_{l} \in B_{i}$ thì $k<l$.
2. Gọi $a_{i}, b_{i}$ lần lượt là số phần tử của tập hợp $A_{i}, B_{i}$ với $i=1,2,3, \ldots, m$. Chúng minh rằng
$$
\sum_{i=1}^{m} \frac{1}{C_{a_{i}+b_{i}}^{a_{i}}} \leq 1
$$

Bài 8. Cho tam giác $A B C$ nhọn nội tiếp đường tròn tâm $O$. Đường tròn tâm $I$ đi qua $B$, $C$ lần lượt cắt các tia $B A$, CA tại $E, F$.
1. Giả sử các tia $B F, C E$ cắt nhau tại $D$ và $T$ là tâm đường tròn $(A E F)$. Chứng minh rằng $O T$ || ID.
2. Trên BF, CE lần lượt lấy các điểm $G, H$ sao cho $A G \perp C E, A H \perp B F$. Các đường tròn $(A B F),(A C E)$ cắt $B C$ tai $M, N$ (khác $B, C)$ và cắt EF tại $P, Q$ (khác $E, F)$. Gọi $K$ là giao điểm của $M P, N Q$. Chứng minh rằng DK vuông góc với GH.

Giải

Bài 1.

Cho $n$ là số nguyên dương lớn hơn 1 , ta quy ước gọi một số nguyên dương a được gọi là thặng dư chính phương theo modulo $n$ nếu $(a, n)=1$ và tồn tại số nguyên $x$ sao cho $a \equiv x^{2}(\bmod n)$. Trong bài này, dể dơn giản, ta quy ước xét các thặng dư chính phưong nhỏ hơn $n$.
Đặt $s(n)$ là số các số nhỏ hơn $n$ và là thặng dư chính phương theo modulo n. Ta sễ chứng minh hai bổ dề dưới đây:
Bổ đề 1: Cho $p$ là số nguyên tố và $k$ là số nguyên dưong. Khi đó:
1. Nếu $p=2$ thì $s\left(2^{k}\right)=2^{\max (k-3,0)}$.
2. Nếu $p>2$ thì $s\left(p^{k}\right)=\frac{p^{k}-p^{k-1}}{2}$.
Bổ đề $2: s(n)$ là hàm nhân tính.
Thật vậy,
Trước hết, ta biết rằng $s(p)=\frac{p-1}{2}$ với $p$ là số nguyên tố lẻ. Ta sẽ tính $s\left(p^{k}\right)$ với $k \in \mathbb{Z}^{+}$.
Xét một thặng dư chính phương $a$ của $p$, khi đó tồn tại $x$ sao cho
$$
a \equiv x^{2}(\bmod p)
$$
Dặt $a=x^{2}+p q$ thì hiển nhiên
$$
a \equiv x^{2}+p q\left(\bmod p^{k}\right) \Leftrightarrow a-p q \equiv x^{2}\left(\bmod p^{k}\right)
$$
và khi đó, ta có $p^{k-1}$ cách chọn $q$ để các số $a-p q$ là các thặng dư chính phưong $\bmod p^{k}$.
Suy ra
$$
s\left(p^{k}\right)=p^{k-1} s(p)=\frac{p^{k}-p^{k-1}}{2}
$$
Xét số nguyên tố $p=2$, với $k=1,2,3$, dể dàng kiểm tra được $s\left(2^{k}\right)=1$.
Ta xét $k \geq 4$, tưong tự trên, ờ bước chọn $q$, ta chỉ có 2 cách nên $s\left(2^{k}\right)=$ $2 s\left(2^{k-1}\right)$. Từ đó bằng quy nạp, ta có được
$$
s\left(2^{k}\right)=2^{k-3}, k \geq 4
$$
Tiếp theo, xét hai số $a, b$ nguyên dương và $(a, b)=1 .$ Gọi $A$ là tập họp các thặng dư chính phương theo modulo $a b$ và $B$ là tập hợp các số là thặng dư chính phưong chung của $a, b$.
Nếu $x \in A$ thì tổn tại $y$ sao cho $x \equiv y^{2}(\bmod a b)$. Rō ràng khi đó,
$$
x \equiv y^{2} \quad(\bmod a), x \equiv y^{2} \quad(\bmod b)
$$

(chú ý rằng nếu $x>a$, ta có thể chọn $x^{\prime}$ sao cho $x^{\prime}<a$ và $x \equiv x^{\prime}(\bmod a)$; tương tự với $b$ ). Do đó, $x \in B$, tức là $x \in A \Rightarrow x \in B$ nên $|A| \leq|B|$.
Tiếp theo, xét $x \in B$. Khi đó tồn tại $r, s$ sao cho $x \equiv r^{2}(\bmod a), x \equiv s^{2}$ $(\bmod b)$. Theo định lý thặng dư Trung Hoa, tổn tại số nguyên $z$ sao cho
$$
z \equiv r(\bmod a), z \equiv s(\bmod b)
$$
Khi đó
$$
x \equiv z^{2} \quad(\bmod a), x \equiv z^{2} \quad(\bmod b)
$$
nên
$$
x-z^{2}: a b \text { hay } x \equiv z^{2}(\bmod a b)
$$
Do đó: $x \in A$, tức là $x \in B \Rightarrow x \in A$ nên $|A| \geq|B|$.
Từ đây ta có
$$
|A|=|B| \text { hay } s(a) s(b)=s(a b)
$$
Vậy $s(n)$ là hàm nhân tính.
Các bổ đề đều được chứng minh.
Trở lại bài toán, ta thấy rằng
$$
2016=2^{5} \cdot 3^{2} \cdot 7
$$
Rō ràng bài toán yêu cầu đếm số thặng dư chính phương theo modulo 2016. Theo bổ dề 2 thì
$$
s(2016)=s\left(2^{5}\right) s\left(3^{2}\right) s(7)
$$
Theo bổ đề 1 thì
$$
s\left(2^{5}\right)=2^{2}=4, s\left(3^{2}\right)=\frac{3^{2}-3}{2}=3, s(7)=\frac{7-1}{2}=3
$$
Do đó, số các số $a$ cần tìm là $4 \cdot 3 \cdot 3=36$.

Bài 2.

1) Dự đoán dấu bằng xảy ra khi $a=1, b=2, c=3, d=4$ nên ta có các đánh giá sau
$$
\left\{\begin{array}{l}
a^{2}+1 \geq 2 a \\
b^{2}+4 \geq 4 b \\
c^{2}+9 \geq 6 c \\
d^{2}+16 \geq 8 d
\end{array}\right.
$$
Do đó, ta có
$$
\begin{aligned}
&24(a+b+c+d) \leq 3\left(d^{2}+16\right)+4\left(c^{2}+9\right)+6\left(b^{2}+4\right)+12\left(a^{2}+1\right) \\
&=3 d^{2}+4 c^{2}+6 b^{2}+12 a^{2}+120 \\
&=3\left(a^{2}+b^{2}+c^{2}+d^{2}\right)+\left(a^{2}+b^{2}+c^{2}\right)+2\left(a^{2}+b^{2}\right)+6 a^{2}+120 \\
&\leq 3 \cdot 30+14+2 \cdot 5+6 \cdot 1+120=240
\end{aligned}
$$
Suy ra $a+b+c+d \leq 10$.
2) Ta có:
$$
16 a^{2}+d^{2} \geq 8 a d \text { và } 9 b^{2}+4 c^{2} \geq 12 b c
$$
Từ đó suy ra
$$
\begin{aligned}
&24(a d+b c) \leq 3\left(16 a^{2}+d^{2}\right)+2\left(9 b^{2}+4 c^{2}\right) \\
&=3\left(a^{2}+b^{2}+c^{2}+d^{2}\right)+5\left(a^{2}+b^{2}+c^{2}\right)+10\left(a^{2}+b^{2}\right)+30 a^{2} \\
&\leq 3 \cdot 30+5 \cdot 14+10 \cdot 5+30 \cdot 1=240
\end{aligned}
$$
Suy ra $a d+b c \leq 10$.

Bài 3.

Goi $(*)$ là điều kiện đề bài cho. Trong $(*)$, thay $x=y=0$, ta có
$$
f(-2 f(0))=3 f(0)
$$
Đặt $f(0)=a$ thì $f(-2 a)=3 a$. Trong $(*)$, thay $x=0$ và $y=-2 a$, ta có
$$
f(-2 f(-2 a))=5 a-2 f(-2 a) \Leftrightarrow f(-6 a)=-a
$$

Trong $(*)$, thay $x=-2 a, y=-6 a$, ta có
$$
\begin{aligned}
&f(-2 a-2 f(-6 a))=5 f(-2 a)-4 x-2 f(-6 a) \\
&\Leftrightarrow f(0)=15 a+8 a+2 a \\
&\Leftrightarrow a=25 a \\
&\Leftrightarrow a=0
\end{aligned}
$$
Do đó $f(0)=0$.
Trong $(*)$, thay $y=0$, ta có
$$
f(x)=5 f(x)-4 x \Leftrightarrow f(x)=x
$$
Thử lại ta thấy thỏa.
Vậy hàm số cần tìm chính là
$$
f(x)=x, \forall x \in \mathbb{R}
$$

Bài 4.

1) Gọi $O$ là tâm của đường tròn $k$. Không mât tính tống quát, giả sử tia $\Lambda D$ nằm giữa hai tia $A O, A B$, các trường hợp còn lại tương tự.
Ta có:
$$
\angle I D B=\angle A B C, \angle I D C=\angle A C B
$$
nên
$$
\angle B A C+\angle B D C=\angle B A C+\angle A B C+\angle A C B=180^{\circ}
$$

Do đó, tứ giác $A B D C$ nội tiếp hay $D \in(O)$. Ta thấy
$$
\begin{aligned}
&\angle D A O+\angle O I D \\
&=\angle B A C-(\angle D A B+\angle O A C)+360^{\circ}-\left(90^{\circ}+\angle D I C\right) \\
&=\angle B A C-\left(\angle I C D+90^{\circ}-\angle A B C\right)+270^{\circ}-\angle D I C \\
&=\angle B A C+\angle A B C-(\angle I C D+\angle D I C)+180^{\circ} \\
&=\left(180^{\circ}-\angle A C B\right)-\left(180^{\circ}-\angle I D C\right)+180^{\circ} \\
&=\angle I D C-\angle A C B+180^{\circ}=180^{\circ}
\end{aligned}
$$

Do đó, AOID nội tiếp hay đường tròn $(A I D)$ di qua $O$ cố định.
2) Ta có:
$$
\angle E A C=90^{\circ}-\angle B A C, \angle F A B=90^{\circ}-\angle B A C
$$
nên
$$
\angle E A F=180^{\circ}-2 \angle B A C+\angle B A C=180^{\circ}-\angle B A C
$$
Do đó, góc $\angle E A F$ có số đo không đổi.

Bài 5.

Trước hết, ta chứng minh bổ đề sau:
Giá trị của biểu thức
$$
\frac{1}{1}+\frac{1}{2}+\frac{1}{3}+\cdots+\frac{1}{n}
$$
tiến tới vô cực khi $n \rightarrow+\infty$. Thật vậy,
Xét hàm số $f(x)=\ln (1+x)-x$ với $x>0$. Ta có
$$
f^{\prime}(x)=\frac{1}{1+x}-1<0
$$
nên đây là hàm nghịch biến, suy ra $f(x)<f(0)=0$ hay $\ln (1+x)<$ $x, \forall x>0$. Thay $x$ bởi $\frac{1}{n}$, ta được
$$
\ln \left(1+\frac{1}{n}\right)<\frac{1}{n} \Leftrightarrow \frac{1}{n}>\ln (1+n)-\ln n
$$
Do đó,
$$
\frac{1}{1}+\frac{1}{2}+\frac{1}{3}++\frac{1}{n}>\ln 2-\ln 1+\ln 3-\ln 2+\cdots+\ln (n+1)-\ln n=\ln (n+1)
$$
Vì $\ln (n+1) \rightarrow+\infty$ khi $n \rightarrow+\infty$ nên
$$
\frac{1}{1}+\frac{1}{2}+\frac{1}{3}+\cdots+\frac{1}{n} \rightarrow+\infty
$$
Trở lại bài toán, đặt
$$
y_{n}=\frac{x_{1}+x_{3}+x_{5}+\cdots+x_{2 n-1}}{x_{2}+x_{4}+x_{6}+\cdots+x_{2 n}}
$$
với $n \geq 1$. Ta thấy vì $\frac{1}{n} \in\left(0 ; \frac{\pi}{2}\right)$ nên $\cos \frac{1}{n}>0$, suy ra
$$
x_{n}=\frac{1}{n \cos \frac{1}{n}}>0, n \geq 1
$$

Xét hàm số $f(t)=\frac{t}{\cos t}$ với $t \in\left(0 ; \frac{\pi}{2}\right)$ thì $f^{\prime}(t)=\frac{\cos t+t \sin t}{\cos ^{2} t}>0$ nên đây là hàm đồng biến. Chú ý rằng $x_{n}=f^{2}\left(\frac{1}{n}\right)$, mà $\frac{1}{n}$ là dãy giảm nên $x_{n}$ cũng là dãy giảm.
Suy ra $x_{1}>x_{2}, x_{3}>x_{4}, \ldots, x_{2 n-1}>x_{2 n}$ nên $y_{n}>1$
Ngoài ra, ta cũng có $x_{3}<x_{2}, x_{5}<x_{4}, \ldots, x_{2 n-1}<x_{2 n-2}$ nên
$$
\begin{aligned}
y_{n}<& \frac{x_{1}+\left(x_{2}+x_{4}+\cdots+x_{2 n-2}\right)}{x_{2}+x_{4}+\cdots+x_{2 n}}=\\
& 1-\frac{x_{1}-x_{2 n}}{x_{2}+x_{4}+\cdots+x_{2 n}}<1-\frac{x_{1}}{x_{2}+x_{4}+\cdots+x_{2 n}}
\end{aligned}
$$
Dễ thấy rằng
$$
x_{2}+x_{4}+\cdots+x_{2 n}=\sum_{i=1}^{n} \frac{1}{2 i \cos \frac{1}{2 i}} \geq \sum_{i=1}^{n} \frac{1}{2 i}=\frac{1}{2} \sum_{i=1}^{n} \frac{1}{i}
$$
Theo bổ đề trên thì $\sum_{i=1}^{n} \frac{1}{i}$ tiến tới vô cực nên
$$
\lim \left(x_{2}+x_{4}+\cdots+x_{2 n}\right)=+\infty
$$
Do dó
$$
\lim \left(1-\frac{x_{1}}{x_{2}+x_{4}+\cdots+x_{2 n}}\right)=1-0=1
$$
Theo nguyên lý kẹp, ta có $\lim x_{n}=1$.

Bài 6.

Đặt $X=x-1, Y=y+1$, thay vào, ta có
$$
\begin{aligned}
&\left\{\begin{array}{l}
X^{2}+Y^{2}=b \\
Y-1=(X+1)^{2}+(2 a+1)(X+1)+a^{2}
\end{array}\right. \\
&\Leftrightarrow\left\{\begin{array}{l}
X^{2}+Y^{2}=b \\
Y=X^{2}+(2 a+3) X+a^{2}+2 a+3
\end{array}\right.
\end{aligned}
$$
Ta đưa về tìm điều kiện của $b$ để tồn tại $a$ mà hệ trên có nghiệm $(X, Y)$. Do
$$
Y-(X+2)=X^{2}+2(a+1) X+(a+1)^{2}=(X+a+1)^{2} \geq 0
$$

nên $Y \geq X+2$. Suy ra $Y-X \geq 2>0$, tức là $(X-Y)^{2} \geq 4$. Ta có
$$
b=X^{2}+Y^{2}=\frac{(X-Y)^{2}+(X+Y)^{2}}{2} \geq \frac{(Y-X)^{2}}{2} \geq 2
$$
Mặt khác, với $b \geq 2$, nếu chọn $X=-(a+1)$ thì có $Y=X+2=1-a$. Khi đó, ta có
$$
X^{2}+Y^{2}=(a+1)^{2}+(a-1)^{2}=2\left(a^{2}+1\right)=b
$$
Như thế, với $a$ thỏa mãn $2\left(a^{2}+1\right)=b$ thì hệ có nghiệm là
$$
(X, Y)=(-a-1,1-a)
$$
Dễ dàng thấy rằng do $b \geq 2$ nên luôn tồn tại $a$ như thế.
Vậy các giá trị cần tìm của $b$ là $b \geq 2$.

Bài 7.

1) Giả sử ngược lại, tồn tại 2 cặp $\left(A_{i}, B_{i}\right)$ và $\left(A_{j}, B_{j}\right)$ thỏa mãn điểu kiện đề bài đã cho.
Vì $i \neq j$ nên theo giả thiết,
$$
\left|A_{i} \cap B_{j}\right| \geq 1,\left|A_{j} \cap B_{i}\right| \geq 1
$$
Đặt $x_{r} \in A_{i} \cap B_{j}, x_{s} \in A_{j} \cap B_{i}$ với $1 \leq r, s \leq n$ thì:
– Do $x_{r} \in B_{j}$ nên với mọi $x_{k} \in A_{j}$, ta đều có $k<r$.
– Do $x_{r} \in A_{i}$ nên với mọi $x_{k} \in B_{i}$, ta đều có $k>r$.

Từ đây suy ra
$$
A_{j} \subset\left\{x_{1}, x_{2}, \ldots, x_{r-1}\right\}, B_{i} \subset\left\{x_{r+1}, x_{r+2}, \ldots, x_{n}\right\}
$$
Điều này cho thấy $A_{j} \cap B_{i}=\varnothing$, mâu thuẫn với giả thiết.
Vậy tồn tại không quá 1 cặp $\left(A_{i}, B_{i}\right)$ thỏa mãn điều kiện đã cho.
2) Gọi $T$ là tập hợp các cách chọn hai dãy
$$
A_{1}, A_{2}, \ldots, A_{m} \text { và } B_{1}, B_{2}, \ldots, B_{m}
$$
thỏa mãn điều kiện là: với mỗi $i, j \in\{1,2,3, \ldots, n\}, A_{i} \cap B_{j}=\varnothing$ nếu và chỉ nếu $i=j$.
Gọi $T_{i} \subset T$ là các cách chọn sao cho sao cho cặp $\left(A_{i}, B_{i}\right)$ thỏa mãn điều kiện là: cặp $\left(A_{i}, B_{i}\right)$ với $i=1,2,3, \ldots, n$ sao cho nếu $x_{k} \in A_{i}$ và $x_{l} \in B_{i}$ thì $x_{k}<x_{l}$ (ở đây ta xét thứ tự ban đầu của các phần tử của $X$ ). (*)
Theo câu 1) thì $T_{i} \cap T_{j}=\varnothing$ với $i \neq j$ nên ta có
$$
\left|T_{1}\right|+\left|T_{2}\right|+\cdots+\left|T_{m}\right|=\left|T_{1} \cup T_{2} \cup \ldots \cup T_{m}\right| \leq T
$$
Tiếp theo, với $1 \leq i \leq m$, xét một tập hợp $S \subset X$ và $|S|=a_{i}+b_{i}$. Khi đó, tương ứng với $S$, có đúng 1 cách chọn $\left(A_{i}, B_{i}\right)$ thỏa mãn tính chất $(*)$ – tức là $A_{i}$ sẽ nhận $a_{i}$ số nhỏ nhất trong tập $S, B_{i}$ là lấy phần còn lại.
Trong khi đó, nếu không có điều kiện $(*)$, ta có thể chọn tùy ý $C_{a_{i}+b_{i}}^{a_{i}}$ phần tử trong $S$ và $A$ và số còn lại cho $B$.
Do đó, ta có
$$
\left|T_{i}\right|=\frac{|T|}{C_{a_{i}}^{a_{i}}+b_{i}}
$$
với $i=1,2, \ldots, m$. Từ đây suy ra
$$
\sum_{i=1}^{m} \frac{|T|}{C_{a_{i}+b_{i}}^{a_{i}}} \leq|T| \Leftrightarrow \sum_{i=1}^{m} \frac{1}{C_{a_{i}+b_{i}}^{a_{i}}} \leq 1
$$
Ta có đpcm.

Bài 8.

1) Giả sử $E F$ cắt $B C$ ở $L$ và $(T),(O)$ cắt nhau tại $J$ khác $A$. Suy ra $A J$ chính là trục đẳng phương của $(T),(O)$. Do đó $O T \perp A J$.
Khi đó,
$$
L B \cdot L C=L E \cdot L F
$$
nên $L$ thuộc trục đẳng phương của $(T),(O)$. Suy ra $A, J, L$ thẳng hàng. Theo định lý Brocard cho tứ giác $B E F C$ nội tiếp trong đường tròn $(I)$ thì $I$ chính là trực tâm của tam giác $A D L$.
Vì thế nên ID $\perp A L$, mà $O T \perp A J$ nên $I D \| O T$.

2) Dễ dàng thấy rằng $D$ là trực tâm của tam giác $A G H$ nên $A D \perp G H$. Ta sẽ chứng minh rằng $A, D, K$ thẳng hàng.

Ta có $D B \cdot D F=D E \cdot D C$ nên $D$ có cùng phương tích tới 2 đường tròn $(A B F),(A E C)$. Suy ra $A D$ chính là trục đẳng phương của 2 đường tròn này.

Bằng biến đổi các góc nội tiếp, ta thấy rằng
$$
\angle M P Q=\angle M B F=\angle C E F=\angle C N Q
$$
Suy ra $M N P Q$ nội tiếp, dẫn đến $K M \cdot K P=K N \cdot K Q$, tức là $K$ cũng có cùng phương tích tới 2 đường tròn $(A B F),(A E C)$.
Từ đó suy ra $A, D, K$ thẳng hàng. Do đó, $D K$ vuông góc với $G H$.

Đề thi vào lớp 10 chuyên toán Phổ thông Năng khiếu: Năm 2016

ĐỀ BÀI

BÀI 1. 
a) Giải hệ $\left\{\begin{array}{l} (x-2y)(x+my) = m^2-2m-3 \\(y-2x)(y+mx) = m^2-2m-3
\end{array} \right.$ khi $m = -3$ và tìm $m$ để hệ co ít nhất một nghiệm $(x_o, y_o)$ thỏa $x_o > 0, y_o > 0$.
b)  Tìm $a \geq 1$ để phương trình $ax^2 + (1-2a)x + 1-a=0$ có hai nghiệm phân biệt $x_1, x_2$ thỏa $x_2^2 – ax_1 = a^2-a-1$.
BÀI 2.  Cho $x, y$ là hai số nguyên dương mà $x^2 + y^2 + 10$ chia hết cho $xy$.

a) Chứng minh rằng $x, y$ là hai số lẻ và nguyên tố cùng nhau.
b)  Chứng minh $k = \dfrac{x^2+y^2+10}{xy}$ chia hết cho 4 và $k \geq 12$.

BÀI 3.  Biết $x \geq y \geq z, x + y + z =0$ và $x^2 + y^2 + z^2 = 6$.

a) Tính $S = (x-y)^2 + (x-y)(y-z) + (y-z)^2$.
b) Tìm giá trị lớn nhất của $P = |(x-y)(y-z)(z-x)|$.

BÀI 4. Tam giác $ABC$ nhọn có $\angle BAC > 45^o$. Dựng các hình vuông $ABMN, ACPQ$ ($M$ và $C$ khác phía đối với $AB$; $B$ và $Q$ khác phía đối với $AC$). $AQ$ cắt đoạn $BM$ tại $E$ và $NA$ cắt đoạn $CP$ tại $F$.

a) Chứng minh $\triangle ABE \sim \triangle ACF$ và tứ giác $EFQN$ nội tiếp.
b) Chứng minh trung điểm $I$ của $EF$ là tâm đường tròn ngoại tiếp tam giác $ABC$.
c) $MN$ cắt $PQ$ tại $D$, các đường tròn ngoại tiếp các tam giác $DMQ$ và $DNQ$ cắt nhau tại $K$ ($K$ khác $D$), các tiếp tuyến tại $B$ và $C$ của đường tròn ngoại tiếp tam giác $ABC$ cắt nhau tại $J$. Chứng minh các điểm $D, A, K, J$ thẳng hàng.

BÀI 5. Với mỗi số nguyên dương $m$ lớn hơn 1, kí hiệu $s(m)$ là ước nguyên dương lớn nhất của $m$ và khác $m$. Cho số tự nhiên $n > 1$, đặt $n_o = n$ và lần lượt tính các số $n_1 =n_o- s(n_o), n_2 = n_1 – s(n_1), …, n_{i+1} = n_i – s(n_i)$,…. Chứng minh tồn tại số nguyên dương $k$ để $n_k = 1$ và tính $k$ khi $n = 2^{16}.14^{17}$.

Hết

Lời giải. 

Bài 1: 

a) Đây là hệ đối xứng loại 2, nên phương pháp giải là lấy (1) – (2) để có thừa số $x-y$, từ đó giải tiếp.

Chú ý xét trường hợp và điều kiện $x_o > 0, y_o > 0$ để biện luận. Những dạng toán này chú ý tính toán cẩn thận và xét đầy đủ các trường hợp.

b) Là bài dạng  biểu thức nghiệm không đối xứng, có nhiều cách, có thể tính nghiệm theo $m$ từ đó suy ra $m$.

Lời giải.

a) Khi $m = -3$ ta có hệ:

$\left\{\begin{array}{l} (x-2y)(x-3y)=12 \\(y-2x)(y-3x) = 12 \end{array} \right.$

$\Leftrightarrow \left\{\begin{array}{l} x^2-5xy+6y^2=12 (1)\\y^2-5xy+6x^2 = 12(2) \end{array} \right.$

Lấy (1) – (2) ta có $5(y^2-x^2) = 0 \Leftrightarrow x = y, x = -y$.
Với $x= y$ thế vào (1) ta có $x^2 =6 \Leftrightarrow x = \sqrt{6}, y = \sqrt{6}$ hoặc $x=-\sqrt{6}, y = -\sqrt{6}$.
Với $x = -y$ thế vào (1) ta có $x^2 = 1 \Leftrightarrow x = 1, x = -1$. Với $x = 1, y = -1$, với $x=-1, y = 1$.
Vậy hệ phương trình có 4 nghiệm.
Hệ có thể viết lại $\left\{\begin{array}{l} x^2+(m-2)xy-2my^2 = m^2-2m-3 (1)\\y^2+(m-2)xy-2mx^2= m^2-2m-3(2) \end{array} \right.$

Lấy (1) – (2) ta có $(2m+1)(y^2-x^2) = 0$.
Xét $m = \dfrac{-1}{2}$ ta có hệ trở thành: $x^2 – \dfrac{5}{2}xy + y^2 + \dfrac{7}{4}=0$, có nghiệm $ (\dfrac{5+\sqrt{2}}{2},2)$ thỏa đề bài.
Xét $m \neq \dfrac{-1}{2}$ ta có $x = y$ hoặc $x = -y$.

Trường hợp $x = -y$ không thỏa đề bài.
Trường hợp $x = y$, thế vào (1) ta có:

$-(m+1)x^2 = m^2-2m-3 = (m+1)(m-3)$.
Nếu $m = -1$ ta có $(x-2y)(x-y) = 0, (y-2x)(y-x) = 0$ có nghiệm thỏa đề bài, chỉ cần chọn $x=1, y=1$.
Nếu $m \neq -1$ ta có $x^2 = 3-m$ để có nghiệm $x_o = y_o > 0$ thì $m < 3$.

Khi đó phương trình có nghiệm $x_0 = \sqrt{3-m}, y_o = \sqrt{3-m}$ thỏa đề bài.

Kết luận $m = \dfrac{-1}{2}, m = -1$ và $m < 3$.

b) Điều kiện để phương trình có hai nghiệm phân biệt $\Delta = (1-2a)^2-4a(1-a) = 8a^2-8a+1 > 0$.
Theo định lý Viete ta có $x_1 + x_2 = \dfrac{2a-1}{a}$, suy ra $ax_1 + ax_2 = 2a – 1$. Suy ra $ax_1 = 2a-1-ax_2$.
Kết hợp giả thiết ta có $x_2^2+ax_2-2a+1=a^2-a-1
\Leftrightarrow x_2^2+ax_2-a^2-a+2=0
\Leftrightarrow ax_2^2+a^2x_2-a^3-a^2+2a=0$ (1).
Mà $x_2$ là nghiệm của phương trình nên ta có $ax_2^2+(1-2a)x_2+1-a = 0 (2)$.
Lấy (1) – (2) ta có $(a^2+2a-1)x_2 = a^3+a^2-3a+1$, mà $a \geq 1$ nên $a^2 + 2a – 1 \neq 0$, suy ra $x_2 = a-1$.
Thế vào phương trình (1) ta có $(a-1)^2+a(a-1)-a^2-a+2 = 0 \Leftrightarrow a=1, a=3$.
Thử lại ta nhận hai giá trị $a = 1, a=3$.

Bài 2.

a) Giả sử trong hai số $x, y$ có một số chẵn, vì vai trò $x, y$ như nhau nên có thể giả sử $x$ chẵn. Suy ra $x^2 + y^2 + 10$ chia hết cho 2, suy ra $y$ chẵn. Khi đó $x^2 + y^2 + 10$ chia hết cho 4, suy ra 10 chia hết cho 4 vô lý.
Vậy trong hai số đều là số lẻ.
Đặt $d= (x,y)$, $x= d.x’, y = d.y’$ ta có $x^2 + y^2 + 10 = d^2(x’^2 + y’^2) + 10$ chia hết cho $d^2x’y’$. Suy ra 10 chia hết cho $d^2$. Suy ra $d= 1$. Vậy $x, y$ nguyên tố cùng nhau.

b) Đặt $x = 2m + 1, y = 2n + 1$, suy ra $k = \dfrac{4(m^2+m+n^2+n+3}{(2m+1)(2n+1)}$, ta có $4, (2m+1).(2n+1)$ nguyên tố cùng nhau. Suy ra $m^2 + n^2 +m+n+3$ chia hết cho $(2m+1)(2n+1)$. Từ đó ta có $k$ chia hết cho 4. Chứng minh $k \geq 12$ bằng hai cách.
Cách 1: Ta có $x^2 + y^2 + 10 = kxy$.
Nếu trong hai số $x, y$ có một số chia hết cho 3, giả sử $x$ chia hết cho 3. Ta có $y^2 + 10$ chia hết cho 3 vô lý vì $y^2 $ chia 3 dư 0 hoặc dư 1.
Vậy $x, y$ không chia hết cho 3, suy ra $x^2 + y^2 + 10$ chia hết cho 3 và $3, xy$ nguyên tố cùng nhau. Do đó $k$ chia hết cho 3.
Do đó $k$ chia hết cho 12, vậy $k\geq 12$.
Cách 2: Xét $k=4$ ta có $x^2 + y^2 + 10 = 4xy$ () $\Leftrightarrow (x-2y)^2 = 3y^2 – 10$.
Ta có $(x-2y)^2$ chia 3 dư 0 hoặc 1 mà $3y^2-10$ chia 3 dư 2, nên phương trình (
) không có nghiệm nguyên dương.
Xét $k=8$ ta có $x^2 + y^2 + 10 = 8xy (*)\Leftrightarrow (x-4y)^2 = 15y^2 -10$.
Ta có $(x-4y)^2$ chia 3 dư 0 hoặc 1 mà $15y^2-10$ chia 3 dư 2 nên (**) không có nghiệm nguyên dương.
Vậy $k \geq 12$.

Bài 3. Bài này là bài bdt khó, nhưng câu a đã gợi ý để làm câu b, chú ý các bdt phụ quan trọng.

a) Ta có $(x+y+z)^2 = x^2+y^2+z^2 + 2(xy+yz+xz)$. Suy ra $xy + yz + xz = -3$.
Ta có $S = (x-y)^2 + (x-y)(y-z) + (y-z)^2 $

$= x^2 -2xy+y^2+xy-y^2+yz-xz+y^2-2yz + z^2$

$= x^2+y^2+z^2-yx-yz-xz = 9$.

b) Ta có thể chứng minh trực tiếp không qua câu a) như sau:

$(x-y)(y-z) \leq \dfrac{1}{3}((x-y)^2+(x-y)(y-z) + (y-z)^2) = 3$. Suy ra $P \leq 3|x-z|$.
Ta có $|x-z| \leq \sqrt{2(x^2+z^2)}\leq \sqrt{2(x^2+y^2+z^2)}= \sqrt{12}$. Suy ra $P \leq 3\sqrt{12} = 6\sqrt{3}$.
Đẳng thức xảy ra khi $x = \sqrt{3}, y =0, z = -\sqrt{3}$.

Vậy giá trị lớn nhất của P là $6\sqrt{3}$ khi $x = \sqrt{3}, y =0, z = -\sqrt{3}$

Ngoài ra ta có thể áp dụng câu a: Đặt $a = x-y, b = y-z$ ta có $a^2+b^2+ab = 9$, cần tìm giá trị lớn nhất của $P = ab(a+b)$.

Áp dụng $ab \leq \dfrac{1}{4} (a+b)^2$ và $a^2+b^2+ab \geq \dfrac{3}{4} (a+b)^2$. Ta có điều cần chứng minh.

Bài 4. Đây là bài hình khó và dài, các em chú ý hình vẽ cụ thể là góc, vẽ hình chính xác. 

Tránh dùng các kiến thức cấp 3: phương tích trục đẳng phương,…

a) Ta có $\angle EAB + \angle BAC = 90^\circ, \angle FAC + \angle BAC = 90^\circ$. Suy ra $\angle EAB = \angle FAC$.
Mặt khác có $\angle ABE = \angle ACF = 90^\circ$. Suy ra $\triangle ABE \backsim \triangle ACF$.
Suy ra $AE\cdot AC = AF\cdot AB$ mà $ AC = AQ, AB = AN$. Suy ra $AE\cdot AQ = AN\cdot AF$. Suy ra tứ giác $QNEF$ nội tiếp.
b) Cách 1: Gọi $T$ là giao điểm của $MB$ và $CP$. Ta có $ABTC$ nội tiếp và $AT$ là đường kính của đường tròn ngoại tiếp tam giác $ABC$. Mặt khác ta có $AF|| ET, AE|| FT$ nên $AETF$ là hình bình hành. Suy ra trung điểm $EF$ cũng là trung điểm $AT$. Do đó trung điểm $I$ của $EF$ là tâm đường tròn ngoại tiếp tam giác $ABC$.
Cách 2: Xét hình thang $AEBF$, gọi $X$ là trung điểm của $AB$ khi đó $IX$ thuộc đường trung bình của hình thang, suy ra $IX || BE$ hay $IX$ vuông góc $AB$ vậy $IX$ là trung trực của đoạn $AB$. Chứng minh tương tự thì $I$ cũng thuộc trung trực đoạn $AC$. Vậy $I$ là tâm ngoại tiếp của tam giác $ABC$.

b) $DA$ cắt $EF$ tại $K’$ ta có $\angle NFK’ = \angle NQA$ (vì $NQFE$ nội tiếp). Mà $\angle NQA = \angle NDA$(vì $AQDN$ nội tiếp). Suy ra $\angle NDA = \angle AFK’$.
Suy ra $NDFK’$ nội tiếp. Chứng minh tương tự ta có $DQK’E$ nội tiếp.
Do đó $K’$ là giao điểm của đường tròn ngoại tiếp hai tam giác $DQM$ và $DPN$. Vậy $K’ \equiv K$. Suy ra $D, A, K$ thẳng hàng.
Ta có $\angle BKE = \angle EAB = \angle CAF = \angle CKF$. Suy ra $\angle BKC = 180^\circ – 2 \angle BKE = 2(90^\circ – \angle EAB) = 2\angle BAC = \angle BIC$. Suy ra $BKIC$ nội tiếp. Mà $IBJC$ nội tiếp, suy ra và $JB = JC$ nên $\angle BKJ = \angle CKJ$. Hay $KJ$ là phân giác $\angle BKC$.
Mặt khác $\angle BKA = 180^\circ – \angle AEB = 180^\circ – \angle AFC = \angle AKC$. Suy ra tia đối của tia $KA$ cũng là phân giác của $\angle BKC$. Do đó $A, K, J$ thẳng hàng.
Vậy 4 điểm $D, A, K, J$ thẳng hàng.

Bài 5. Đây là bài toán lạ và khá hay, sử dụng đơn biến.

Ta có $s(n_i) < n_i$, suy ra $n_i – s(n_i) \geq 1$. Suy ra $n_{i+1} \geq 1$. Do đó $n_i \geq 1$ với mọi $i = 1, 2, …$.
Mặt khác $n_{i+1} = n_i – s(n_i) < n_i$ với mọi $i$. Suy ra $n=n_o > n_1 > n_2 > …>…$.
Nếu không tồn tại $n_k$ để $n_k = 1$ ta xây dựng được dãy vô hạn các số nguyên dương giảm và nhỏ hơn $n$ (vô lý) vì số các số nhỏ hơn $n$ là bằng $n-1$.
Vậy tồn tại $k$ sao cho $n_k = 1$.
Với $n=2^{16}.14^{17} = 2^{33}.7^{17}$, ta có $n_1 = 2^{33}7^{17} – 2^{32}.7^{17}= 2^{32}.7^{17}$.\
$n_2 = 2^{31}.7^{17}$.
Tiếp tục ta có $n_{33} = 7^{17}$.
Đặt $m_o= 7^{17}$ ta có $m_1 = 6.7^{16}$, $m_2 = 3.7^{16}, m_3 = 2.7^{16}, m_4 = 7^{16}$. Tương tự ta có $m_8 = 7^{15}$,…,$m_{68} = 7^0 = 1$.
Vậy $k = 33 + 68 = 101$.

Đề thi và đáp án tuyển sinh vào lớp 10 PTNK không chuyên 2016

Đề thi vào lớp 10 trường Phổ thông Năng khiếu năm 2016

Bài 1. Biết $a$ và $b$ là các số dương, $a \neq b$ và

$$\left(\dfrac{a(a-4b)+b(b+2a) }{a+b}\right):\left[\left(\dfrac{a\sqrt{a}+b\sqrt{b}}{\sqrt{a}+\sqrt{b}} -\sqrt{ab}\right) \left( \dfrac{a\sqrt{a}-b\sqrt{b}}{\sqrt{a}-\sqrt{b}} + \sqrt{ab}\right)\right] = 2016$$

Tính $S=a+b$.

Giải

Ta có $\dfrac{a\left( a – 4b \right) + b\left( b + 2a \right)}{a + b} = \dfrac{a^2 – 2ab + b^2}{a + b} = \dfrac{\left( a – b \right)^2}{a + b}$

$\dfrac{{a\sqrt a + b\sqrt b }}{{\sqrt a + \sqrt b }} – \sqrt {ab} = \dfrac{{\left( {\sqrt a + \sqrt b } \right)\left( {a – \sqrt {ab} + b} \right)}}{{\sqrt a + \sqrt b }} – \sqrt {ab} = {\left( {\sqrt a – \sqrt b } \right)^2}$

$\dfrac{{a\sqrt a – b\sqrt b }}{{\sqrt a – \sqrt b }} + \sqrt {ab} = {\left( {\sqrt a + \sqrt b } \right)^2}$

Do đó $2016 = \dfrac{(a-b)^2}{a+b}:\left[(\sqrt{a}-\sqrt{b})^2(\sqrt{a}+\sqrt{b})^2\right] = \dfrac{1}{a+b}$.

Suy ra $a + b = \dfrac{1}{2016}$.

Bài 2.

a) Giải phương trình $x\sqrt{x+5}=2x^2-5x$.

b) Giải hệ phương trình $\left\{\begin{array}{l} (\sqrt{y}+x-3)(y+\sqrt{x})=0\\ x^2+y=5 \end{array}\right.$

Giải

a) Điều kiện $x \geq -5$.

Ta có $x\sqrt{x+5}=2x^2-5x \Leftrightarrow x(\sqrt{x+5}-2x+5)=0 \Leftrightarrow x= 0 (n), \sqrt{x+5}= 2x-5$.

Ta có $\sqrt{x+5}= 2x-5 \Leftrightarrow x+5 = (2x-5)^2 (x \geq \dfrac{5}{2}) \Leftrightarrow 4x^2-21x+20 = 0 \Leftrightarrow x = 4 \ (n), x = \dfrac{5}{4}\ (l)$.

Vậy $S = \{0, 4\}$.

b) Điều kiện $x \geq 0, y \geq 0$.

Ta có $(1) \Leftrightarrow y + \sqrt{x}=0, \sqrt{y}+x-3=0$.

Với $y + \sqrt{x}=0$ mà $y\geq 0$ nên $x = y = 0$ (không thỏa $(2)$).

Với $\sqrt{y}+x-3=0$. Đặt $a =\sqrt{y}$ ta có $a+x = 3 (3)$; $a^2+x^2=5 (4)$.

Từ $(3)$ ta có $a = 3-x$, thế vào $(4)$ ta có

$x^2+(3-x)^2=5 \Leftrightarrow 2x^2-6x+4=0 \Leftrightarrow x = 1,x=2$.

Với $x = 1$ ta có $y = 4$.

Với $x = 2$ ta có $y = 1$.

Vậy hệ phương trình có $2$ nghiệm $(x,y)$ là $(1,4)$ và $(2,1)$.

Bài 3. Cho phương trình $\dfrac{(x+1)(x^2+mx+2m+14)}{\sqrt{x}} = 0 \ (1)$.

a) Giải phương trình (1) khi $m = -8$.

b) Tìm $m$ để phương trình $(1)$ có $2$ nghiệm phân biệt $x_1,x_2$ sao cho:

$$\sqrt{x_2^2+(m+1)x_2+2m+14} = 3 – \sqrt{x_1}$$

Giải

a) Điều kiện $x > 0$.

Khi $m = -8$ ta có phương trình:

$\dfrac{(x+1)(x^2-8m-2)}{\sqrt{x}} = 0 \Leftrightarrow x^2-8x – 2 = 0$ (do $x+1 > 0$)

$\Leftrightarrow x = 4+3\sqrt{2} \ (n), \ x=4-3\sqrt{2} \ (l)$.

Vậy phương trình có một nghiệm $x = 4 +3\sqrt{2}$.

b) Phương trình $(1)$ tương đương $x^2+mx+2m+14 = 0$ $(2)$.

Để $(1)$ có $2$ nghiệm phân biệt thì $(2)$ có hai nghiệm phân biệt dương, tương đương $\Delta = m^2-4(2m+14) > 0, S = -m > 0, P = 2m + 14 >0 $ $(*)$

Khi đó $x_1 + x_2 = -m, x_1x_2 = 2m+14$ và $x_2$ là nghiệm nên $x_2^2+mx_2+2m+14 = 0$

Suy ra $x_2^2+(m+1)x_2 +2m+14 = x_2$.

Do đó $\sqrt{x_2^2+(m+1)x_2+2m+14} = 3 – \sqrt{x_1} $

$\Leftrightarrow \sqrt{x_1}+\sqrt{x_2}=3 \Leftrightarrow x_1 + x_2 +2\sqrt{x_1x_2}=9 $

$\Leftrightarrow 2\sqrt{2m+14}=9+m \Leftrightarrow 4(2m+14) = m^2+18m+81 $

$\Leftrightarrow m^2 +10m+25 = 0 \Leftrightarrow m = -5 \ (n)$ vì thỏa $(*)$.

Kết luận $m = -5$.

Bài 4.

a) Ông An định cải tạo một mảnh vườn hình chữ nhật có chiều dài bằng $2,5$ chiều rộng. Ông thấy rằng nếu đào một cái hồ có mặt hồ là hình chữ nhật thì sẽ chiếm mất $3\%$ diện tích mảnh vườn, còn nếu giảm chiều dài $5m$ và tăng chiều rộng $2m$ thì mặt hồ là hình vuông và diện tích mặt hồ giảm được $20m^2$. Hãy tính các cạnh của mảnh vườn.

b) Lớp $9A$ có $27$ học sinh nam và $18$ học sinh nữ. Nhân dịp sinh nhật bạn $X$ (là một thành viên của lớp), các bạn trong lớp có rất nhiều món quà tặng $X$. Ngoài ra mỗi bạn nam của lớp làm $3$ tấm thiệp và mỗi bạn nữ xếp $2$ hoặc $5$ con hạc để tặng bạn $X$. Biết số tấm thiệp và số con hạc bằng nhau, hỏi bạn $X$ là nam hay nữ?

Giải

a) Gọi chiều dài và chiều rộng của hồ là $x, y\ (m)$.

Ta có $x-5 = y + 2$ (1) và $xy – (x-5)(y+2) = 20$ $(2)$.

Từ $(1)$ suy ra $x =y + 7$, thế vào $(2)$ ta có

$y(y+7) -(y+2)^2 =20 \Leftrightarrow 3y = 24 \Leftrightarrow y = 8, x = 15$.

Suy ra diện tích hồ là 120$m^2$.

Gọi chiều rộng của mảnh vườn là $a$.

Ta có chiều dài là $2,5a$ và diện tích là $2,5a^2$.

Ta có phương trình $3\%2,5a^2=120 \Leftrightarrow a = 40$.

Vậy kích thước mảnh vườn là $40 \times 100$.

b) Gọi $x$ là số bạn nữ tặng $2$ con hạc, $y$ là số bạn nữa tặng $5$ con hạc.

  • Giả sử bạn $X$ là nam, ta có hệ phương trình $26.3 = 2x+5y, x + y = 18$.

Giải ra được $y= 14, x = 4$ (thỏa).

  • Giả sử bạn $X$ là nữ, ta có hệ $27.3 = 2x+5y, x + y = 17$

Suy ra $y = \dfrac{47}{3}$ (loại vì $y$ là số nguyên).

Vậy bạn $X$ là nam.

Bài 5. Tam giác $ABC$ đều có tâm $O$,$AB = 6a$ và các điểm $M, N$ lần lượt thuộc các cạnh $AB, AC$ mà $AM = AN = 2a$. Gọi $I, J, K$ lần lượt là trung điểm của $BC, AC$ và $MN$.

a) Chứng minh các điểm $M, N, B, C$ cùng thuộc một đường tròn T. Tính diện tích tứ giác $BMNC$ theo $a$.

b) Tính bán kính đường tròn ngoại tiếp tam giác $IJK$. Chứng minh đường tròn đường kính $NC$ tiếp xúc với $AI$.

c) $AE$ tiếp xúc với đường tròn $T$ tại $E$ ($E, B$ cùng phía đối với $AI$).Gọi $F$ là trung điểm $OE$, tính số đo $\angle AFJ$.

Giải

a) Ta có $AM = AN = 2a$,$\angle MAN = 60^\circ$ nên tam giác $AMN$ đều.

Suy ra $\angle AMN = 60^\circ = \angle ACB$. Suy ra $BMNC$ nội tiếp.

Ta có $MN ||BC$, $AK \bot MN, AI \bot BC$. Suy ra $A, K, I$ thẳng hàng.

$AI = AC \sin \angle ACB = 3a \sqrt{3}$, $AK = AN . \sin \angle ANM = a\sqrt{3}$.  Suy ra $IK = 2a\sqrt{3}$.

Do đó $S_{BMNC} = \dfrac{1}{2}IK(MN+BC) = 8a^2\sqrt{3}$.

b) Ta có $OJ \bot AC$, $NJ = AJ-AN=a, NK = \dfrac{1}{2}MN=a$.

Suy ra $\Delta OJN = \Delta OKN$, suy ra $OJ = OK$, tương tự ta có $OJ = OI$.

Tam giác $IJK$ nội tiếp đường tròn tâm $O$ bán kính $OI = a\sqrt{3}$.

Gọi $P$ là trung điểm của $CN$. Ta có $KNCI$ là hình thang, và $OP$ là đường trung bình.

Suy ra $OP = \dfrac{1}{2}(KN+CI) = 2a = PN = PC$.

Suy ra $O$ thuộc đường tròn đường kính $CN$ mà $PO||KN$ nên $PO \bot KI$.

Suy ra $KI$ là tiếp tuyến của đường tròn đường kính $CN$.

c) Ta có $\angle AEM = \angle ABE$. Suy ra $\Delta AEM \backsim \Delta ABE$, suy ra $AE^2=AM.AB = 12a^2$.

Suy ra $AE = 2a\sqrt{3}= AO$. Suy ra tam giác $AEO$ cân tại $A$.

Do đó $\angle AFO = 90^\circ$, suy ra $AFOJ$ nội tiếp. Suy ra $\angle AFJ = \angle AOJ = 60^\circ$.

Đề thi và đáp án thi chọn đội dự tuyển trường PTNK năm học 2016-2017

ĐỀ BÀI

Bài 1. Cho $x,y,z$ là các số thực dương thoả mãn $x+y+z=1$. Chứng minh rằng:
$$\dfrac{x^4}{x^3+y^2+z^2}+\dfrac{y^4}{y^3+z^2+x^2}+\dfrac{z^4}{z^3+x^2+y^2}\ge \dfrac{1}{7}.$$

Bài 2. Tìm tất cả các hàm số $f:N^* \rightarrow  N^*$ thoả mãn đồng thời các điều kiện:

i)  $ f(mn)=f(m)f(n) \forall m,n \in N^* $.
ii) $f(m)+f(n)$ chia hết cho $m+n$, $\forall m,n \in N^* $.
iii) $f(2017)=2017^3$.

Bài 3.  Cho đường tròn $(O)$ và dây cung $AB$ cố định. $C$ là một điểm thay đổi trên cung lớn $AB$ sao cho tam giác $ABC$ nhọn. Gọi $I,I_a,I_b$ lần lượt là tâm đường tròn nội tiếp, tâm đường tròn bàng tiếp $\angle BAC$ và $\angle ABC$ của tam giác $ABC$.
a) Gọi $M$ đối xứng với $I$ qua $O$. Chứng minh rằng tam giác $MI_{a}I_{b}$ cân.
b) Gọi $H,K$ lần lượt là hình chiếu của $I_a,I_b$ trên $OI$. Đường thẳng qua $H$ vuông góc với $BI_a$ và đường thẳng qua $K$ vuông góc với $AI_b$ cắt nhau tại $P$. Chứng minh rằng $P$ thuộc một đường cố định khi $C$ thay đổi.

Bài 4.  Cho $S$ là tập hợp khác rỗng và $A_1,A_2,\ldots,A_m\ (m\ge 2)$ là $m$ tập con của $S$. Gọi $\mathcal T$ là tập hợp gồm tất cả các tập hợp $A_i\Delta A_j\ (1\le i,j \le m$). Chứng minh rằng $|\mathcal T| \ge m$. \medskip

(Ký hiệu $A\Delta B=(A\backslash B)\cup (B\backslash A)$ là hiệu đối xứng của hai tập hợp $A,B$).

 

Đề thi và đáp án tuyển sinh vào lớp 10 TPHCM 2016

I. ĐỀ 

Bài 1. Giải các phương trình và hệ phương trình sau:
a) $x^2-2\sqrt{5}x+5=0$
b) $4x^4-5x^2-9=0$
c) $2x+5y=-1$ và $3x-2y=8 $
d) $x(x+3)=15-(3x-1)$.

Bài 2.
a) Vẽ đồ thị (P) của hàm số $y = \dfrac{-x^2}{4}$ và đường thẳng (D): $y = \dfrac{x}{2}-2$ trên cùng một hệ trục tọa độ.
b) Tìm tọa độ giao điểm của (P) và (D) ở câu trên bằng phép tính.
Bài 3.
a) Thu gọn biểu thức $A = \dfrac{2-\sqrt{3}}{1+\sqrt{4+2\sqrt{3}}} + \dfrac{2+\sqrt{3}}{1-\sqrt{4-2\sqrt{3}}}$
b) Ông Sáu gửi một số tiến vào ngân hàng theo mức lãi suất tiết kiệm với kù hạn 1 năm là 6$\%$. Tuy nhiên sau thời hạn một năm ông Sáu không đến nhận tiền lãi mà để thêm một năm nữa mới lãnh. Khi đó số tiền lãi có được sau năm đầu tiên sẽ được ngân ghàng cộng dồn vào số tiền gửi ban đầu để thành số tiền gửi cho năm kế tiếp với mức lãi suất cũ. Sau 2 năm ông Sáu nhận được số tiền là 112.360.000 đồng kể cả gốc lẫn lãi. Hỏi ban đầu ông Sáu đã gửi bao nhiêu tiền?
Bài 4. Cho phương trình $x^2 – 2mx + m – 2= 0 $(1) ($x$ là ẩn số.)

a) Chứng minh phương trình (1) luôn có 2 nghiệm phân biệt với mọi giá trị $m$.
b) Định $m$ để hai nghiệm $x_1, x_2$ của phương trình (1) thỏa mãn : $(1+x_1)(2-x_2) + (1+x_2)(2-x_1) = x_1^2+x_2^2+2 $
Bài 5. Cho tam giác $ABC$ $(AB < AC) $ có ba góc nhọn. Đường trong tâm $O$ đường kính $BC$ cắt các cạnh $AC, AB$ lần lượt tại $D, E$.
Gọi $H$ là giao điểm của $BD$ và $CE$; $F$ là giao điểm của $AH$ và $BC$.
a) Chứng minh $AF \bot BC$ và $\angle AFD = \angle ACE$.
b) Gọi $M$ là trung điểm của $AH$. Chứng minh $BD \bot OD$ và 5 điểm $M, D, O, F, E$ cùng thuộc một đường tròn.
c) Gọi $K$ là giao điểm của $AH$ và $DE$. Chứng minh $MD^2 = MK.MF$ và $K$ là trực tâm của tam giác $MBC$.
d) Chứng minh $\dfrac{2}{FK} = \dfrac{1}{FH} + \dfrac{1}{FA}$.

II. ĐÁP ÁN

Bài 1. Giải các phương trình và hệ phương trình sau:
a) $x^2 – 2\sqrt{5}x + 5=0$
$\Delta ‘= 0
x_1=x_2 = \sqrt{5}$.
b) $4x^4 – 5x^2 -9 =0$
Đặt $t=x^2 \ge 0$
Phương trình trở thành: $4t^2 – 5t -9=0$
$a-b+c =0$.
$\Rightarrow t_1 =-1$ (loại) và $t_2 = \dfrac{9}{4}$ (nhận)
Với $t=\dfrac{9}{4} \Rightarrow x= \pm \dfrac{3}{2}$
c) $2x + 5y =-1 $ và $3x-2y=8$
$ \Leftrightarrow 4x+ 10y =-2 $ và $15x -10y =40 $
$ \Leftrightarrow x=2$ và $y=-1$.
d) $x(x+3) = 15 – (3x-1) $
$\Leftrightarrow x^2 + 6x -16 =0$
$\Leftrightarrow x_1 =2$; $x_2 = -8$.

Bài 2.
a) Lưu ý: $(P)$ đi qua $O(0;0)$, $( \pm 2 ;-1)$, $\pm 4; -4 )$
$(D)$ đi qua $(2;-1)$, $(0;-2)$

Đồ thị:
b) Phương trình hoành độ giao điểm của $(P)$ và $(D)$ là:
$-\dfrac{x^2}{4}= \dfrac{x}{2}-2 $
$\Leftrightarrow x^2 + 2x -8 =0 $
$\Leftrightarrow x=-4$ hoặc $x=2$

$y(-4) = -4$, $y(2) = -1$
Vậy tọa độ giao điểm của $(P)$ và $(D)$ là $(-4;-4)$, $(2;-1)$.
Bài 3.
a) $A=\dfrac{2-\sqrt{3}}{1+\sqrt{4+2\sqrt{3}}}+ \dfrac{2+\sqrt{3}}{1-\sqrt{4-2\sqrt{3}}} $
$= \dfrac{2-\sqrt{3}}{1+ \left( 1+ \sqrt{3} \right) } + \dfrac{2+\sqrt{3}}{1- \left( \sqrt{3}-1 \right) } $
$= \dfrac{\left( 2+ \sqrt{3} \right) ^2 + \left( 2- \sqrt{3} \right) ^2}{\left( 2+\sqrt{3} \right) \left( 2- \sqrt{3}\right) } $
$=14$

b) Gọi số tiền ban đầu ông Sáu gửi là: $x$ (đồng)
Số tiền vốn và lãi sau năm thứ nhất là: $x+x \cdot 6 \% = 1,06 x$
Số tiền vốn và lãi sau năm thứ hai là: $1,06x + 1,06x \cdot 6\% = 1,06^2 \cdot x$
Theo đề ta được phương trình:\ $1,06^2 \cdot x = 112.360.000 \Rightarrow x= 100.000.000$ (đồng)
Bài 4.

a) $x^2 -2mx +m-2 =0$
$\Delta ‘= m^2 -m+2 = \left( m- \dfrac{1}{2} \right) ^2 + \dfrac{7}{4} >0, \; \forall m$
Do đó phương trình luôn có hai nghiệm phân biệt.
b) Theo Viet, ta có:

$S= x_1+ x_2 = 2m $ và  $P = x_1 \cdot x_2 = m-2$

$\left( 1+ x_1 \right) \left( 2-x_2 \right) + \left( 1+ x_2 \right) \left( 2- x_1 \right) = x_1^2 + x_2^2 +2 $
$\Leftrightarrow 2+ x_1 + x_2 = \left( x_1 + x_2 \right) ^2 $
$\Leftrightarrow 2+ 2m = 4m^2 $
$\Leftrightarrow m=1 $ hoặc $m= \dfrac{-1}{2}$
Bài 5.


a)

  • $\angle BEC = \angle BDC = 90^\circ $
    $\Rightarrow $ $CE$ và $BD$ là hai đường cao của tam giác $ABC$
    $\Leftrightarrow $ $H$ là trực tâm của tam giác $ABC$
    $\Rightarrow $ $AH$ là đường cao của tam giác $ABC$
    $\Rightarrow AF \bot BC$.
  • Tứ giác $HFCD$ nội tiếp ($\angle HFC + \angle HDC = 180^\circ$)
    $\Rightarrow \angle AFD = \angle ACE$

b)

  • $\angle MAD = \angle MDA$ và $\angle ODC = \angle OCD $
    Mà $\angle FAC + \angle FCA = 90^\circ
    \Rightarrow \angle MDA + \angle ODC = 90^\circ
    \Rightarrow \angle MDO = 90^\circ \Rightarrow MD \bot OD $
  • Chứng minh tương tự: $ME \bot OE$
  •  3 điểm $E$, $F$, $D$ cùng nhìn $MO$ dưới 1 góc $90^\circ$
    $\Rightarrow $ 5 điểm $M$, $D$, $O$, $F$, $E$ cùng thuộc đường tròn đường kính $MO$

c)

  • $MD$ là tiếp tuyến của đường tròn tâm $O$
    $\Rightarrow \angle MDE = \angle DCE$
    mà $\angle AFD = \angle ACE$ nên $\angle MDK = \angle MFD$
    Vậy $\triangle MDK \backsim \triangle MFD \Rightarrow MD^2 = MK \cdot MF$
  •  $MC$ cắt $(O)$ tại $L$
  • $\triangle MDL \backsim \triangle MCD \Rightarrow MD^2 = ML \cdot MC
    \Rightarrow MK \cdot MF = ML \cdot MC
    \Rightarrow \triangle MLK \backsim \triangle MFC
    \Rightarrow \angle KLM = \angle MFC =90^\circ
    \Rightarrow KL \bot MC$
    Mà $BL \bot MC$ (góc nội tiếp chắn nửa đường tròn)
    $\Rightarrow $ $B$, $K$, $L$ thẳng hàng
    $\Rightarrow$ $K$ là trực tâm $\triangle MBC$.

d)

  • $FH \cdot FA = FB \cdot FC$ ($\triangle BFH \backsim \triangle AFC$)
  • $FK \cdot FM = FB \cdot FC$ ($\triangle BFK \backsim \triangle MFC$)
    $\Rightarrow FH \cdot FA = FK \cdot FM
    \Rightarrow 2FH \cdot FA = 2 FK \cdot FM = FK ( FA + FH )
    \Rightarrow \dfrac{2}{FK} = \dfrac{1}{FH} + \dfrac{1}{FA}$.